Ch 20 Robbins Flashcards

1
Q

A 24-year-old man is awakened at night because of severe lower abdominal pain that radiates to the groin. The pain is very intense and comes in waves. The next morning, he notices blood in his urine. He has no underlying illnesses and has been healthy all his life. On physical examination, he is afebrile and has a blood pressure of 110/70 mm Hg. Laboratory studies show serum Na+, 142 mmol/L; K+, 4 mmol/L; Cl−, 96 mmol/L; CO2, 25 mmol/L; glucose, 74 mg/dL; creatinine, 1.1 mg/dL; calcium, 9.1 mg/dL; and phosphorus, 2.9 mg/dL. Urinalysis shows a pH of 7; specific gravity of 1.020; and no protein, glucose, ketones, or nitrite. The patient is advised to drink more water. He likes iced tea and consumes large quantities over the course of a hot summer. He continues to have similar episodes. Which of the following substances is most likely to be increased in his urine?

□ (A) Calcium oxalate
□ (B) Cystine
□ (C) Magnesium ammonium phosphate 
□ (D) Mucoprotein
□ (E) Uric acid
A

(A) This patient has ureteral colic from the passage of a stone down the ureter. About 70% of all renal stones are composed of calcium oxalate crystals. Patients with these stones tend to have hypercalciuria without hypercalcemia. Uric acid stones and cystine stones are radiolucent and tend to form in acidic urine. Cystine stones are rare. Triple phosphate (magnesium ammonium phosphate) stones tend to occur in association with urinary tract infections, particularly infections caused by urease-positive bacteria, such as Proteus. Mucoproteins may coalesce into hyaline casts, which are too small to produce signs and symptoms.

How well did you know this?
1
Not at all
2
3
4
5
Perfectly
2
Q

A 47-year-old man has had a decreased urine output over the past 10 days. On physical examination, he is afebrile. Urinalysis shows 1+ proteinuria, 4+ hematuria, urobilinogen, and no glucose or ketones. Microscopic examination of the urine shows few WBCs and some RBCs with RBC casts. A renal biopsy is done, and the light microscopic appearance of a PAS-stained specimen is shown in the figure. What is the most likely clinical course in this patient?

□ (A) Acute renal failure that is reversible with supportive therapy
□ (B) Slowly developing renal failure that is unresponsive to corticosteroid treatment
□ (C) Rapidly progressive renal failure accompanied by hemoptysis
□ (D) Stable clinical course with intermittent hematuria
□ (E) Fever, leukocytosis, and endotoxic shock

A

(C) The figure shows glomeruli with epithelial crescents that are the morphologic correlates of rapidly progressive glomerulonephritis. Patients with this condition rapidly develop renal failure. One cause of rapidly progressive renal failure is Goodpasture syndrome, in which anti–glomerular basement membrane antibodies damage the glomeruli and the pulmonary alveoli. Damage to the alveoli results in hemoptysis. Acute tubular necrosis is potentially reversible. Focal segmental glomerulosclerosis is typically nonresponsive to corticosteroids. IgA nephropathy can have intermittent hematuria. Acute pyelonephritis is accompanied by fever and leukocytosis.

How well did you know this?
1
Not at all
2
3
4
5
Perfectly
3
Q

A 63-year-old man has noted increasing back pain for 7 months. He has had three respiratory tract infections with Streptococcus pneumoniae within the past year. On examination, he has pitting edema to his thighs. Laboratory studies show total serum protein, 9.6 g/dL; albumin, 3.5 g/dL; creatinine, 3 mg/dL; urea nitrogen, 28 mg/dL; and glucose, 79 mg/dL. Urinalysis shows proteinuria of 4 g/24 hr, but no glucosuria or hematuria. Abdominal CT scan shows enlarged kidneys without cysts or masses. A renal biopsy specimen shows deposits of amorphous pink material within glomeruli, interstitium, and arteries with H&E stain. Which of the following diseases is he most likely to have?

□ (A) Analgesic nephropathy
□ (B) Diabetes mellitus
□ (C) Membranous glomerulonephritis
□ (D) Multiple myeloma
□ (E) Systemic lupus erythematosus 
□ (F) Wegener granulomatosis
A

(D) There is a large amount of serum globulin, back pain from lytic lesions, immunosuppression with recurrent infections, and amyloid deposition enlarging the kidneys, all consistent with multiple myeloma. This AL amyloid deposition occurs in 6% to 24% of myeloma cases. Analgesic nephropathy can lead to tubulointerstitial nephritis and papillary necrosis. His serum glucose is not in the range for diabetes mellitus, and the pink deposits seen with nodular or diffuse glomerulosclerosis are not amyloid. The pink-staining, thickened capillary loops of membranous glomerulonephritis represent immune deposits, not amyloid. Systemic lupus erythematosus can result in immune deposits to produce “wire loop” thickening of glomerular capillaries.

How well did you know this?
1
Not at all
2
3
4
5
Perfectly
4
Q

A 58-year-old woman dies of a cerebral infarction. Laboratory findings before death included serum urea nitrogen level of 110 mg/dL and creatinine level of 9.8 mg/dL. At autopsy, the kidneys are small (75 g) and have a coarsely granular surface appearance. Microscopic examination shows sclerotic glomeruli, a fibrotic interstitium, tubular atrophy, arterial thickening, and scattered lymphocytic infiltrates. Which of the following clinical findings was most likely reported on the patient’s medical history?

□ (A) Rash
□ (B) Hypertension 
□ (C) Hemoptysis 
□ (D) Lens dislocation 
□ (E) Pharyngitis
A

(B) These findings describe end-stage renal disease, the appearance of which is similar regardless of the cause (e.g., vascular disease or glomerular disease). With advanced renal destruction, hypertension almost always supervenes, even if it was absent at the onset of renal disease. Many such cases are referred to as “chronic glomerulonephritis” for want of a better term. A rash might have preceded the postinfectious glomerulonephritis. Hemoptysis occurs in Goodpasture’s syndrome. Lens dislocation is a feature of Alport syndrome. Pharyngitis with group A streptococcal infection may precede postinfectious glomerulonephritis.

How well did you know this?
1
Not at all
2
3
4
5
Perfectly
5
Q

For the past 6 months, a 72-year-old woman has noticed a slowly enlarging mass on the urethra. The mass causes local pain and irritation and is now bleeding. Physical examination shows a 2.5-cm warty, ulcerated mass protruding from the external urethral meatus. There are no lesions on the labia or vagina. A biopsy specimen of the lesion is most likely to identify which of the following?

□ (A) Embryonal rhabdomyosarcoma 
□ (B) Leiomyoma
□ (C) Papilloma
□ (D) Squamous cell carcinoma 
□ (E) Syphilitic chancre
A

(D) Carcinoma of the urethra is uncommon. It tends to occur in older women and is locally aggressive. An embryonal rhabdomyosarcoma (sarcoma botryoides) is a rare tumor that occurs in children. Benign tumors, such as a leiomyoma or papilloma, are typically well circumscribed and do not ulcerate. Syphilis produces indurated, painless lesions, rather than ulcerated, warty masses.

How well did you know this?
1
Not at all
2
3
4
5
Perfectly
6
Q

A 72-year-old man with Alzheimer disease dies of pneumonia. The gross appearance of the right kidney at autopsy is shown in the figure. The left kidney is normal in size, with a smooth cortical surface and a single 0.6-cm, fluid-filled cyst. The appearance of the right kidney is most suggestive of renal injury from which of the following?

□ (A) Ureteral obstruction
□ (B) Benign nephrosclerosis 
□ (C) Analgesic abuse
□ (D) Chronic pyelonephritis
□ (E) Diabetes mellitus
A

(A) The ureteral, pelvic, and calyceal dilation results from long-standing obstruction leading to hydroureter and hydronephrosis. With benign nephrosclerosis, the kidneys become smaller and develop granular surfaces, but there is no dilation. The scarring that accompanies analgesic nephropathy or chronic pyelonephritis can be marked; it is associated with significant loss of renal parenchyma, but not with pelvic dilation. There are many renal complications of diabetes mellitus, mostly from vascular, glomerular, or interstitial injury, but there is no obstruction. In some patients, diabetes is complicated by a neurogenic bladder, and this can lead to functional obstruction. In such cases, both kidneys and ureters would be affected.

How well did you know this?
1
Not at all
2
3
4
5
Perfectly
7
Q

A 25-year-old man has a 5-year history of celiac sprue. Several days after a mild upper respiratory infection, he begins passing dark red-brown urine. The dark urine persists for the next 3 days and then becomes clear and yellow, only to become red-brown again 1 month later. There are no remarkable findings on physical examination. Urinalysis shows a pH of 6.5; specific gravity 1.018; 3+ hematuria; 1+ proteinuria; and no glucose or ketones. Microscopic examination of the urine shows RBCs and no WBCs, casts, or crystals. A 24-hour urine protein level is 200 mg. A renal biopsy specimen from the glomeruli of this patient is most likely to show which of the following alterations?

□ (A) Subepithelial electron-dense deposits
□ (B) Granular staining of the basement membrane by anti-IgG antibodies
□(C) Mesangial IgA staining by immunofluorescence
□ (D) Diffuse proliferation and basement membrane thickening
□ (E) Thrombosis in the glomerular capillaries

A

(C) IgA nephropathy, also known as Berger’s disease, can explain the presence of recurrent hematuria in a young adult. Nephrotic syndrome is not present, and mesangial IgA deposition is characteristic. The initial episode of hematuria usually follows an upper respiratory infection. IgA nephropathy occurs with increased frequency in patients with celiac disease. Granular staining of basement membrane with IgG antibodies denotes immune complex deposition, which may occur in postinfectious glomerulonephritis. The subepithelial deposits are seen on electron microscopy. Patients with these changes have nephritic syndrome. Diffuse proliferation and basement membrane thickening denote membranoproliferative glomerulonephritis. In this condition, IgG and C3 are deposited in the glomeruli. Glomerular capillary thrombosis is typical of hemolytic-uremic syndrome.

How well did you know this?
1
Not at all
2
3
4
5
Perfectly
8
Q

A 7-year-old boy is recovering from impetigo. Physical examination shows a few honey-colored crusts on his face. The crusts are removed, and a culture of the lesions grows group A Streptococcus pyogenes. He is treated with antibiotics. One week later, he develops malaise with nausea and a slight fever and passes dark brown urine. Laboratory studies show a serum antistreptolysin O titer of 1:1024. Which of the following is the most likely outcome?

□ (A) Development of rheumatic heart disease
□ (B) Chronic renal failure
□ (C) Lower urinary tract infection
□ (D) Complete recovery without treatment
□ (E) Progression to crescentic glomerulonephritis

A

(D) These findings are characteristic of poststreptococcal glomerulonephritis. The strains of group A streptococci that cause poststreptococcal glomerulonephritis differ from the strains that cause rheumatic fever. Most children with poststreptococcal glomerulonephritis recover, although perhaps 1% develop a rapidly progressive glomerulonephritis. Progression to chronic renal failure occurs more frequently in affected adults. A urinary tract infection is not likely to accompany poststreptococcal glomerulonephritis because the organisms that caused the immunological reaction are no longer present when symptoms of glomerulonephritis appear.

How well did you know this?
1
Not at all
2
3
4
5
Perfectly
9
Q

A 28-year-old, previously healthy man suddenly develops severe abdominal pain and begins passing red urine. There are no abnormalities on physical examination. Urinalysis shows a pH of 7; specific gravity 1.015; 1+ hematuria; and no protein, glucose, or ketones. The patient is given a device to use in straining the urine for calculi. The next day, the patient recovers a 0.3-cm stone that is sent for analysis. The chemical composition is found to be calcium oxalate. What underlying condition is most likely to be present?

□ (A) Gout
□ (B) Acute cystitis
□ (C) Diabetes mellitus
□ (D) Primary hyperparathyroidism 
□ (E) Idiopathic hypercalciuria
A

(E) Calcium oxalate stones are the most common type of urinary tract stone. Approximately 50% of patients with calcium oxalate stones have increased excretion of calcium without hypercalcemia. The basis of hypercalciuria is unclear. Most uric acid stones are formed in acidic urine and are not related to gout. It is thought that these patients have an unexplained tendency to excrete acidic urine. At low pH, uric acid is insoluble, and stones form. Infections can predispose to the formation of magnesium ammonium phosphate stones. Diabetes mellitus is an uncommon cause of urinary tract lithiasis; although infections are more common in diabetics, most are not caused by urea-splitting bacteria. Hyperparathyroidism predisposes affected individuals to form stones containing calcium, but few patients with urinary tract stones have this condition.

How well did you know this?
1
Not at all
2
3
4
5
Perfectly
10
Q

A 56-year-old man has had back pain and has passed dark-colored urine for the past month. On physical examination, there is right costovertebral angle tenderness. Urinalysis shows a pH of 6; specific gravity 1.015; 2+ hematuria; and no protein, glucose, or ketones. Microscopic examination of the urine shows numerous RBCs, few WBCs, and no casts or crystals. The figure shows the representative gross appearance of the renal lesion. Which of the following laboratory findings is most likely to be reported?

□ (A) Elevated serum cortisol level 
□ (B) Elevated hematocrit
□ (C) Ketonuria
□ (D) Decreased creatinine clearance
□ (E) Increased plasma renin activity
A

(B) The figure shows a renal cell carcinoma. About 5% to 10% of these tumors secrete erythropoietin, giving rise to polycythemia. Other substances can be secreted—among them corticotropin (adrenocorticotropic hormone), resulting in hypercortisolism in Cushing’s syndrome—but these cases are encountered less frequently than polycythemia. Ketonuria is a feature of type 1 diabetes mellitus, which is not associated with the development of renal neoplasms. Renal cell carcinomas are usually unilateral, and typically they do not destroy all of a kidney. There is no significant loss of renal function, and the serum urea nitrogen and creatinine levels are not elevated. Hypertension from hyperreninemia can occur in patients with some renal cell carcinomas, although this is uncommon. Ketonuria is seen in patients with decreased caloric intake and with type 1 diabetes mellitus. Renal cell carcinomas are usually unilateral and involve part of the kidney; renal failure is unlikely.

How well did you know this?
1
Not at all
2
3
4
5
Perfectly
11
Q

A 15-year-old boy has been passing dark-colored urine for the past month. On physical examination, he has bilateral sensorineural hearing loss and corneal erosions. Urinalysis shows a pH of 6.5; specific gravity 1.015; 1+ hematuria; 1+ proteinuria; and no ketones, glucose, or leukocytes. The serum creatinine level is 2.5 mg/dL, and the urea nitrogen level is 24 mg/dL. A renal biopsy specimen shows tubular epithelial foam cells by light microscopy. By electron microscopy, the glomerular basement membrane shows areas of attenuation, with splitting and lamination of lamina densa in other thickened areas. What is the most likely diagnosis?

□ (A) Acute tubular necrosis
□ (B) Berger disease
□ (C) Membranous glomerulonephritis 
□ (D) Diabetic nephropathy
□ (E) Alport syndrome
A

(E) Alport syndrome is a form of hereditary nephritis. Hematuria is the most common presenting feature, but proteinuria is often present and may be in the nephrotic range. Patients progress to chronic renal failure in adulthood. Most patients have an X-linked dominant pattern of inheritance, but autosomal dominant and autosomal recessive pedigrees also exist. The foamy change in the tubular epithelial cells and ultrastructural alterations of the basement membrane are characteristic features. The genetic defect results from mutation in the gene for the α-5 chain of type IV collagen. Acute tubular necrosis follows ischemic or toxic injuries to the kidney and does not involve glomeruli. Berger disease, or IgA nephropathy, is a form of glomerulonephritis that does not produce tubular epithelial changes. Membranous glomerulonephritis generally produces a nephrotic syndrome and deposition of immune complexes in glomerular basement membrane. Nodular and diffuse glomerulosclerosis are typical changes in diabetic nephropathy.

How well did you know this?
1
Not at all
2
3
4
5
Perfectly
12
Q

A 32-year-old man has developed a fever and skin rash over the past 3 days. Five days later, he has increasing malaise and visits his physician. On physical examination, the maculopapular erythematous rash on his trunk has nearly faded away. His temperature is 37.1°C, and blood pressure is 135/85 mm Hg. Laboratory studies show a serum creatinine level of 2.8 mg/dL and blood urea nitrogen level of 29 mg/dL. Urinalysis shows 2+ proteinuria; 1+ hematuria; and no glucose, ketones, or nitrite. The leukocyte esterase result is positive. Microscopic examination of urine shows RBCs and WBCs, some of which are eosinophils. What is the most likely cause of this patient’s condition?

□ (A) Urinary tract infection 
□ (B) Congestive heart failure 
□ (C) Antibiotic use
□ (D) Streptococcal pharyngitis 
□ (E) Poorly cooked ground beef
A

(C) These findings are typical of drug-induced interstitial nephritis. Various drugs can cause this condition, including sulfonamides, penicillins, cephalosporins, the fluoroquinolone antibiotics ciprofloxacin and norfloxacin, and the antituberculous drugs isoniazid and rifampin. Acute tubulointerstitial nephritis also can occur with use of thiazide and loop diuretics, cimetidine, ranitidine, omeprazole, and nonsteroidal anti-inflammatory drugs. The disease manifests about 2 weeks after the patient begins to use the drug. Elements of type I (increased IgE) and type IV (skin test positivity to drug haptens) hypersensitivity are present. WBCs, but not eosinophils, may be present in the urine of a patient with a urinary tract infection. Congestive heart failure can lead to acute tubular necrosis, but it is not associated with a rash or proteinuria. Poststreptococcal glomerulonephritis could account for the proteinuria and hematuria seen in this patient, but not for the rash because the strains of group A β-hemolytic streptococci that cause a skin infection precede by weeks the development of glomerulonephritis. Hemolytic-uremic syndrome can occur after ingestion of strains of Escherichia coli that may be present in ground beef.

How well did you know this?
1
Not at all
2
3
4
5
Perfectly
13
Q

After eating a cheeseburger, French fries, and ice cream for dinner one night, a 6-year-old girl develops nausea, mild abdominal cramping, and a slight fever. Three days later, her parents notice that she is passing dark stools and dark urine and appears fatigued and weak. On physical examination, she has a temperature of 37.9°C, pulse of 88/min, respirations of 18/min, and blood pressure of 140/90 mm Hg. Scattered petechiae are present on the extremities. Laboratory findings show a serum creatinine level of 2.2 mg/dL and urea nitrogen level of 20 mg/dL. Urinalysis shows a pH of 6; specific gravity 1.016; 2+ hematuria; and no protein or glucose. A renal biopsy specimen shows small thrombi within glomerular capillary loops. Which of the following diseases is most likely to produce these findings?

□ (A) Postinfectious glomerulonephritis 
□ (B) Wegener granulomatosis
□ (C) Hereditary nephritis
□ (D) Hemolytic-uremic syndrome 
□ (E) IgA nephropathy
A

(D) Hemolytic-uremic syndrome is one of the most common causes of acute renal failure in children. It most commonly occurs after ingestion of meat infected with verocytotoxin-producing Escherichia coli, most often serotype O157:H7. The toxin damages endothelium, reducing nitric oxide, promoting vasoconstriction and necrosis, and promoting thrombosis. With supportive therapy, most patients recover in a few weeks, although perhaps one fourth progress to chronic renal failure. Postinfectious glomerulonephritis occurs several weeks after an infection, usually with group A β-hemolytic streptococci. Wegener granulomatosis is a vasculitis that most often occurs in adults. Hereditary nephritis may occur in childhood; it is progressive and is not related to vascular disease. An IgA nephropathy most often occurs in young adults; it is not accompanied by vascular changes.

How well did you know this?
1
Not at all
2
3
4
5
Perfectly
14
Q

The parents of a 6-year-old girl notice that she has become increasingly lethargic over the past 2 weeks. On examination by the physician, she has puffiness around the eyes. Her temperature is 36.9°C, and blood pressure is 100/60 mm Hg. Laboratory findings show a serum creatinine level of 0.7 mg/dL and urea nitrogen level of 12 mg/dL. Urinalysis shows a pH of 6.5; specific gravity 1.011; 4+ proteinuria; and no blood or glucose. The 24-hour urine protein level is 3.8 g. The child’s condition improves after glucocorticoid therapy. Which of the following findings by electron microscopy is most likely to characterize this disease process?

□ (A) Subepithelial electron-dense humps
□ (B) Reduplication of glomerular basement membrane
□ (C) Areas of thickened and thinned basement membrane
□ (D) Increased mesangial matrix
□ (E) Effacement of podocyte foot processes

A

(E) A child with nephrotic syndrome and no other clinical findings is most likely to have lipoid nephrosis, also called minimal change disease. The term minimal change disease reflects the paucity of pathologic findings. There is fusion of foot processes, which can be seen only by electron microscopy. Subepithelial electron-dense humps represent immune complexes and are seen in postinfectious glomerulonephritis. Variability of basement membrane thickening may be seen in Alport syndrome. The mesangial matrix is expanded in some forms of glomerulonephritis (e.g., IgA nephropathy) and other diseases, such as diabetes mellitus, but not in minimal change disease.

How well did you know this?
1
Not at all
2
3
4
5
Perfectly
15
Q

A 25-year-old woman experiences sudden onset of fever, malaise, and nausea. On physical examination, her temperature is 38.2°C, pulse is 85/min, respirations are 18/min, and blood pressure is 140/90 mm Hg. A routine urinalysis shows 1+ proteinuria, 4+ hematuria, and no ketones or glucose. RBC casts are seen on microscopic examination of the urine. A renal biopsy is performed, and light microscopic examination shows marked glomerular hypercellularity with neutrophils in glomerular capillary loops. Immunofluorescence microscopy shows granular deposition of IgG and C3 in glomerular capillary basement membranes. Electron microscopy shows electron-dense subepithelial “humps.” What is the most likely diagnosis?

□ (A) Goodpasture syndrome
□ (B) Systemic amyloidosis
□ (C) Membranous glomerulonephritis 
□ (D) Diabetes mellitus
□ (E) Postinfectious glomerulonephritis
A

(E) Postinfectious glomerulonephritis is one of many causes of a nephritic syndrome characterized by hematuria and RBC casts. Most children recover completely, but one in six adults may progress to chronic renal failure. Some cases may occur after a streptococcal pharyngitis (poststreptococcal glomerulonephritis). In other cases, such as this one, the preceding infection is so mild that patients give no history. Goodpasture syndrome also may produce a nephritic syndrome, but there is linear deposition of antibody in the glomerular basement membrane. Amyloidosis of the kidney mainly produces proteinuria without hematuria, as does membranous glomerulonephritis. Nodular and diffuse glomerulosclerosis are characteristic of diabetic nephropathy.

How well did you know this?
1
Not at all
2
3
4
5
Perfectly
16
Q

A 77-year-old man has had increasing difficulties with urination for the past 2 years. He has difficulty starting and stopping the urine stream. On physical examination, his temperature is 37°C, and blood pressure is 130/85 mm Hg. The figure shows the representative gross appearance of the bladder. Which of the following laboratory findings is most likely to be reported in this patient?

□ (A) Positive ANA test result
□ (B) Urine culture positive for Mycobacterium tuberculosis
□ (C) Hemoglobin concentration of 22.5 g/dL
□ (D) Schistosoma haematobium eggs in urine
□ (E) Prostate-specific antigen level of 5 ng/mL

A

(E) This patient has bladder hypertrophy resulting from outlet obstruction. In an older man, this type of obstruction is most often caused by prostatic enlargement resulting from hyperplasia or carcinoma. Mild elevations in the prostate- specific antigen (PSA) level may occur in patients with prostatic hyperplasia, and greater increases in PSA suggest carcinoma. Autoimmune conditions may be associated with interstitial cystitis, but cystitis does not cause bladder neck obstruction. Bladder outlet obstruction can increase the risk of infection, typically with bacterial organisms such as Escherichia coli, not Mycobacterium tuberculosis. Polycythemia can be the result of a paraneoplastic syndrome, but urothelial malignancies are unlikely to produce this finding; renal cell carcinoma is a more likely cause. Schistosomiasis leads to chronic inflammation and scarring.

How well did you know this?
1
Not at all
2
3
4
5
Perfectly
17
Q

A 50-year-old woman with diabetic nephropathy receives a renal allograft. An episode of acute cellular rejection requires an increase in immunosuppressive therapy. She develops dysuria. On examination, she has suprapubic pain on palpation. A urinalysis shows hematuria. Cystoscopy is performed, and 3- to 4-cm soft, yellow, slightly raised mucosal plaques are seen. Biopsy specimens of these lesions are taken and microscopically show mucosal infiltration by foamy macrophages with abundant PAS-positive cytoplasmic granules and small, laminated mineralized concretions. Which of the following organisms is most likely to be found in her urine?

□ (A) Adenovirus
□ (B) Candida albicans
□ (C) Chlamydia trachomatis 
□ (D) Escherichia coli
□ (E) Schistosoma haematobium
A

(D) This woman has a peculiar form of cystitis known as malacoplakia, when macrophages have reduced phagocytic function, and the concretions within macrophages are known as Michaelis-Gutman bodies. Malacoplakia is a reaction to chronic bacterial infections, usually Escherichia coli and Proteus species, and often in the setting of immunosuppression. The most common organism associated with cases of acute cystitis is E. coli. The other organisms listed are uncommon causes for cystitis and for malacoplakia.

How well did you know this?
1
Not at all
2
3
4
5
Perfectly
18
Q

A 26-year-old man is involved in a motor vehicle accident and sustains acute blood loss. He is hypotensive for several hours before paramedical personnel arrive. They stabilize the bleeding and transport him to a hospital, where he receives a transfusion of 3 U of packed RBCs. Over the next week, the serum urea nitrogen level increases to 48 mg/dL, the serum creatinine level increases to 5 mg/dL, and the urine output decreases. He undergoes hemodialysis for the next 2 weeks and then develops marked polyuria, with urine output of 2 to 3 L/day. His recovery is complicated by bronchopneumonia, but renal function gradually returns to normal. The patient’s transient renal disease is best characterized by which of the following histologic features?

□ (A) Glomerular crescents in Bowman space
□ (B) Interstitial lymphocytic infiltrates
□ (C) Arteriolar fibrinoid necrosis
□ (D) Nodular glomerulosclerosis
□ (E) Rupture of tubular basement membrane

A

(E) This patient’s history is typical of ischemic acute tubular necrosis, which is often accompanied by rupture of the basement membrane (tubulorrhexis). An initiating phase that lasts approximately 1 day is followed by a maintenance phase during which progressive oliguria and increasing blood urea nitrogen levels occur, with salt and water overload. This is followed by a recovery phase during which there is a steady increase in urinary output and hypokalemia. Eventually, tubular function is restored. Treatment of this acute renal failure results in recovery of nearly all patients. Crescents suggest a rapidly progressive glomerulonephritis that is unlikely to resolve. Interstitial infiltrates suggest a chronic tubulointerstitial process. Fibrinoid necrosis in arterioles is a feature of malignant nephrosclerosis, a serious condition that produces significant renal damage. Nodular glomerulosclerosis is a feature of diabetic nephropathy and is a progressive condition that leads to chronic renal failure.

How well did you know this?
1
Not at all
2
3
4
5
Perfectly
19
Q

A 60-year-old, previously healthy man sees his physician because he feels feverish and weak. He reports passing dark-colored urine on several occasions during the past month, but has no urinary frequency, dysuria, or nocturia. On physical examination, his temperature is 37.8°C, and blood pressure is 125/85 mm Hg. A dipstick urinalysis shows 4+ hematuria; 1+ proteinuria; and no glucose or ketones. Which of the following procedures is the most appropriate in management of this patient?

□ (A) Straining of urine for calculi
□ (B) Urine microbiologic culture
□ (C) Abdominal CT scan for renal mass
□ (D) Collection of a 24-hour urine specimen for protein
□ (E) Percutaneous renal biopsy
A

(C) Painless hematuria in an older adult suggests a renal neoplasm. The additional presence of constitutional symptoms, such as fever and weakness, should raise the suspicion of a renal cell carcinoma. Urinary tract calculi usually cause severe, colicky pain when they are passed. Urinary tract infections are not characterized by recurrent hematuria without fever or other signs of acute inflammation. Nephrotic syndrome, which manifests with proteinuria, typically is not associated with hematuria. A renal biopsy has a low yield in a patient without an acute-onset renal disease, and it is an ineffective way of diagnosing tumors.

How well did you know this?
1
Not at all
2
3
4
5
Perfectly
20
Q

A 49-year-old man goes to his physician for a checkup and is found on physical examination to have a blood pressure of 160/110 mm Hg, but no other abnormalities. Laboratory studies show serum glucose of 75 mg/dL, creatinine of 1.3 mg/dL, and urea nitrogen of 20 mg/dL. His plasma renin is elevated. CT angiography shows marked stenosis of his renal arteries. He is treated with an angiotensin-converting enzyme inhibitor. A week later, he has a headache for which he takes ibuprofen. Over the next day, his urine output decreases. A reduction in which of the following chemical mediators most likely caused his reduced urine output?

□ (A) Aldosterone
□ (B) Histamine
□ (C) Nitric oxide
□ (D) Prostaglandin
□ (E) Tumor necrosis factor
A

(D) This patient’s hypertension is due to renal vascular constriction, typical for renal arterial atherosclerosis. In the face of reduced renal blood flow, his glomerular filtration rate (GFR) is maintained by prostaglandin-mediated vasodilation of afferent arterioles and angiotensin II–mediated vasoconstriction of efferent arterioles. The angiotensin-converting enzyme inhibitor decreases efferent arteriolar vasoconstriction and decreases glomerular capillary perfusion pressure. Nonsteroidal anti-inflammatory drugs such as ibuprofen inhibit prostaglandin synthesis and lead to vasoconstriction that reduces renal blood flow and reduces GFR. Aldosterone is increased with increased renin and angiotensin production and leads to reduced sodium excretion. Histamine is a vasodilator from mast cell granules that plays a role in acute inflammatory processes, but not blood pressure regulation. Tumor necrosis factor plays a role in many inflammatory processes, but not renal blood flow. Nitric oxide is a vasodilator, but does not have a significant effect on capillary blood flow.

How well did you know this?
1
Not at all
2
3
4
5
Perfectly
21
Q

A 50-year-old woman has had fever and flank pain for the past 2 days. On physical examination, her temperature is 38.2°C, pulse is 81/min, respirations are 16/min, and blood pressure is 130/80 mm Hg. Urinalysis shows no protein, glucose, or ketones. The leukocyte esterase test is positive. Microscopic examination of the urine shows numerous polymorphonuclear leukocytes and occasional WBC casts. Which of the following organisms is most likely to be found in the urine culture?

□ (A) Mycobacterium tuberculosis 
□ (B) Mycoplasma hominis
□ (C) Escherichia coli
□ (D) Group A streptococcus 
□ (E) Cryptococcus neoformans
A

(C) The clinical features in this patient are typical of urinary tract infection, and Escherichia coli is the most common cause. The WBCs are characteristic of an acute inflammatory process. The presence of WBC casts indicates that the infection must have occurred in the kidney because casts are formed in renal tubules. Most infections of the urinary tract begin in the lower urinary tract and ascend to the kidneys. Hematogenous spread is less common. Mycobacterium tuberculosis causes the rare “sterile pyuria”; however, renal tuberculosis typically does not manifest as an acute febrile illness. Mycoplasma and Cryptococcus are rare urinary tract pathogens. Group A streptococcus is best known as an antecedent infection to poststreptococcal glomerulonephritis, an immunologically mediated disease in which the organisms are not present at the site of glomerular injury.

How well did you know this?
1
Not at all
2
3
4
5
Perfectly
22
Q

A 58-year-old, relatively healthy man sees his physician for a routine health maintenance examination. Physical examination shows mild hypertension. Laboratory findings show a serum creatinine level of 2.2 mg/dL and urea nitrogen level of 25 mg/dL. Microalbuminuria is present, with excretion of 250 mg/day of albumin. Two years later, he returns for a follow-up visit. He is now hypertensive and has a serum creatinine level of 3.8 mg/dL, urea nitrogen level of 38 mg/dL, and 24-hour urine protein level of 2.8 g. A renal biopsy is done; the light microscopic appearance of a PAS-stained specimen is shown in the figure. Which of the following laboratory findings is most likely to be abnormal in this patient?

□ (A) Anti–glomerular basement membrane antibody 
□ (B) ANA
□ (C) ANCA
□ (D) Antistreptolysin O
□ (E) C3 nephritic factor 
□ (F) Hemoglobin A1c
□ (G) Hepatitis B surface antigen
A

(F) The figure shows nodular and diffuse glomerulosclerosis, a classic lesion in diabetes mellitus. Patients with diabetes mellitus have an elevated level of glycosylated hemoglobin (HbA1c). Patients with type 1 diabetes mellitus may initially have microalbuminuria, which predicts development of future overt diabetic nephropathy. There is progressive loss of renal function. These patients are often hypertensive and have hyaline arteriolosclerosis. The presence of overt proteinuria suggests progression to end-stage renal disease within 5 years. Anti–glomerular basement membrane antibody is seen in Goodpasture syndrome, which manifests as a rapidly progressive glomerulonephritis. The ANA test is positive in
a variety of autoimmune diseases, most typically systemic lupus erythematosus, which can be accompanied by glomerulonephritis. The ANCA test is positive in some forms of vasculitis, such as Wegener’s granulomatosis, which can involve the kidneys. The antistreptolysin O titer is elevated after streptococcal infections, which may cause postinfectious glomerulonephritis. The C3 nephritic factor may be present in type II membranoproliferative glomerulonephritis (dense deposit disease). Some patients with membranous glomerulonephritis have a positive serologic test result for HBsAg.

How well did you know this?
1
Not at all
2
3
4
5
Perfectly
23
Q

A 20-year-old woman, G1, P0, who is in the third trimester, has felt minimal fetal movement. An ultrasound scan shows a markedly decreased amniotic fluid index characteristic of oligohydramnios. She gives birth to a stillborn male fetus at 33 weeks’ gestation. At autopsy, there are deformations resulting from marked oligohydramnios, including flattening of the facies, varus deformities of the feet, and marked pulmonary hypoplasia. Microscopic examination of the liver shows multiple epithelium-lined cysts and a proliferation of bile ducts. Which of the following best describes the appearance of the kidneys in this fetus?

□ (A) Bilaterally enlarged kidneys replaced by 1- to 4-cm, fluid-filled cysts
□ (B) Bilaterally shrunken kidneys with uniformly finely granular cortical surfaces
□ (C) Decreased overall size of the right kidney and normal-sized left kidney
□ (D) Irregular cortical scars in asymmetrically shrunken kidneys with marked calyceal dilation
□ (E) Marked bilateral renal pelvic and calyceal dilation with thinning of the cortices
□ (F) Normal-sized kidneys with smooth cortical surfaces
□ (G) Symmetrically enlarged kidneys composed of small, radially arranged cysts

A

(G) Autosomal-recessive polycystic kidney disease (ARPKD) most often occurs in children and may involve the liver; most cases have mutations in the PKHD1 gene encoding for fibrocystin expressed in kidney, liver, and pancreas. By contrast, autosomal-dominant polycystic kidney disease (ADPKD) manifests with renal failure in adults and involves PKD1 and PKD2 gene mutations encoding for polycystin proteins found in renal tubules. Some less common forms of ARPKD are accompanied by survival beyond infancy, and these patients develop congenital hepatic fibrosis. Enlarged kidneys with 1- to 4-cm cysts are characteristic of ADPKD in adults. Perhaps the most common renal cystic disease seen in fetuses and infants is multicystic renal dysplasia (multicystic dysplastic kidney), in which the cysts and kidneys are variably sized. This disease can be focal, unilateral, or bilateral; however, congenital hepatic fibrosis is not present. Small, shrunken, granular kidneys typify end-stage renal diseases in adults. For oligohydramnios to be present, both kidneys must be affected, not just one. Irregular cortical scars with pelvicaliceal dilation may represent hydronephrosis complicated by infection in chronic pyelonephritis, a process that occurs in adults. Dilation with calyceal thinning can occur with obstructions in utero, such as posterior urethral valves in males or urethral atresia in males or females; liver lesions are not present in these cases. A cause of oligohydramnios other than abnormalities of the urinary tract (e.g., leakage of amniotic fluid with premature and prolonged rupture of membranes) could be present if the kidneys appear normal, but in this case the distinctive finding of congenital hepatic fibrosis points to ARPKD.

How well did you know this?
1
Not at all
2
3
4
5
Perfectly
24
Q

A 65-year-old woman has experienced increasing malaise with nocturia and polyuria for the past year. On physical examination, her blood pressure is 170/95 mm Hg. Urinalysis shows a pH of 7.5; specific gravity 1.010; 1+ proteinuria; and no glucose, blood, or ketones. The tests for leukocyte esterase and nitrite yield positive results, and levels of serum urea nitrogen and serum creatinine are elevated. Her clinical course is characterized by worsening renal failure, and she dies of bronchopneumonia. At autopsy, the kidneys are shrunken but unequal in size, and have deep, irregular surface scars. On sectioning, the calyces underlying the cortical scars are blunted and deformed. What is the most likely cause of renal failure in this patient?

□ (A) Chronic glomerulonephritis
□ (B) Essential hypertension
□ (C) Reflux nephropathy
□ (D) Autosomal-dominant polycystic kidney disease 
□ (E) Systemic lupus erythematosus
A

(C) This description of the gross appearance of the kidney is characteristic of chronic pyelonephritis, caused most often by reflux nephropathy. Typical features include coarse and irregular scarring resulting from ascending infection, blunting and deformity of calyces, and asymmetric involvement of the kidneys. The loss of tubules from scarring gives rise to reduced renal concentrating ability; the patient had polyuria with a low specific gravity of the urine. Chronic glomerulonephritis, benign nephrosclerosis (caused by essential hypertension), and systemic lupus erythematosus produce bilateral symmetric involvement, and the affected kidneys are shrunken and finely granular. Autosomal-dominant polycystic kidney disease is characterized by large cysts that replace the renal parenchyma and greatly increase the size of the kidneys bilaterally.

How well did you know this?
1
Not at all
2
3
4
5
Perfectly
25
Q

A 29-year-old woman sees her physician because she has had a fever and sore throat for the past 3 days. On physical examination, her temperature is 38°C. The pharynx is erythematous, with yellowish tonsillar exudate. Group A Streptococcus pyogenes is cultured. She is treated with ampicillin and recovers fully in 7 days. Two weeks later, she develops fever and a rash, and notices a slight decrease in urinary output. Her temperature is 37.7°C, and there is a diffuse erythematous rash on the trunk and extremities. Urinalysis shows a pH of 6; specific gravity 1.022; 1+ proteinuria; 1+ hematuria; and no glucose or ketones. Microscopic examination of the urine shows RBCs and WBCs, including eosinophils, but no casts or crystals. What is the most likely cause of her disease?

□ (A) Deposition of immune complexes with streptococcal antigens
□ (B) Hematogenous dissemination of septic emboli
□ (C) Renal tubular cell necrosis caused by bacterial toxins
□ (D) Hypersensitivity reaction to ampicillin
□ (E) Formation of antibodies against glomerular basement membrane

A

(D) These findings point to an acute drug-induced interstitial nephritis caused by ampicillin. This is an immunological reaction, probably caused by a drug acting as a hapten. Poststreptococcal glomerulonephritis is unlikely to be accompanied by a rash or by eosinophils in the urine. Acute pyelonephritis is an ascending infection; it is uncommonly caused by hematogenous spread of bacteria from other sites. Acute tubular necrosis can cause acute renal failure. It is caused by hypoxia resulting from shock or from toxic injury caused by chemicals such as mercury, and only rarely, if ever, by bacterial toxins. Anti–glomerular basement membrane antibodies occur in Goodpasture’s syndrome, with hemorrhages in lungs as well.

How well did you know this?
1
Not at all
2
3
4
5
Perfectly
26
Q

A 4-year-old girl has complained of abdominal pain for the past month. On physical examination, she is febrile, and palpation of the abdomen shows a tender mass on the right. Bowel sounds are present. Laboratory studies show hematuria without proteinuria. Abdominal CT scan shows a 12-cm, circumscribed, solid mass in the right kidney. A right nephrectomy is done; the gross appearance of the mass is shown in the figure. What is the most likely diagnosis?

□ (A) Angiomyolipoma
□ (B) Interstitial cell tumor
□ (C) Renal cell carcinoma 
□ (D) Transitional cell carcinoma 
□ (E) Wilms tumor
A

(E) Wilms’ tumor is the most common renal neoplasm in children, and one of the most common childhood neoplasms. A complex staging, grading, and molecular analysis formula, and surgery, chemotherapy, and radiation result in a high cure rate. The microscopic pattern of Wilms tumor (nephroblastoma) resembles the fetal kidney nephrogenic zone. Angiomyolipomas may be sporadic or part of the genetic syndrome of tuberous sclerosis. They may be multiple and bilateral and have well-differentiated muscle, adipose tissue, and vascular components. Renomedullary interstitial cell tumors (“medullary fibromas”) are generally less than 1 cm and are incidental findings. Renal cell carcinoma is rare in children, and the most common patterns are clear cell, papillary, and chromophobe. Transitional cell carcinomas arise in the urothelium in adults and microscopically resemble urothelium.

How well did you know this?
1
Not at all
2
3
4
5
Perfectly
27
Q

One week after a mild flulike illness, a 9-year-old boy has an episode of hematuria that subsides within 2 days. One month later, he tells his parents that his urine is red again. On physical examination, there are no significant findings. Urinalysis shows a pH of 7; specific gravity 1.015; 1+ proteinuria; 1+ hematuria; and no ketones, glucose, or urobilinogen. The serum urea nitrogen level is 36 mg/dL, and the creatinine level is 3.2 mg/dL. A renal biopsy specimen shows diffuse mesangial proliferation and electron-dense deposits in the mesangium. Which of the following mechanisms is most likely to produce these findings?

□ (A) Deposition of immune complexes containing IgA
□ (B) Formation of antibodies against type IV collagen
□ (C) Virus-mediated injury to the glomeruli
□ (D) Cytokine-mediated injury to the glomerular capillaries
□ (E) Congenital defects in the structure of glomerular basement membranes

A

(A) Development of recurrent hematuria after a viral illness in a child or young adult is typically associated with IgA nephropathy. In these patients, some defect in immune regulation causes excessive mucosal IgA synthesis in response to viral or other environmental antigens. IgA complexes are deposited in the mesangium and initiate glomerular injury. Antibodies against type IV collagen are formed in Goodpasture syndrome. Although viruses induce IgA synthesis, they do not cause direct glomerular damage. Cytokine-mediated injury can occur in transplant rejection. Defects in the structure of glomerular basement membrane are a feature of hereditary nephritis.

28
Q

The top of the diaper is often noted to be damp on a girl infant. Radiologic imaging with contrast enhancement shows that there is a connection from the bladder to umbilicus. What is the most likely diagnosis?

□ (A) Congenital diverticulum 
□ (B) Exstrophy
□ (C) Persistent urachus 
□ (D) Vesicoureteral reflux 
□ (E) Vitelline duct remnant
A

(C) The embryologic urachus may not become obliterated, leaving a fistulous tract or a cyst between the bladder and abdominal wall at the umbilicus. Congenital diverticula result from either focal failure in formation of bladder musculature or bladder outlet obstruction, and there is no fistulous tract. Exstrophy refers to failure in development of the lower abdominal wall, leaving an open defect to the bladder. Abnormal reflux of bladder contents into the ureter defines vesicoureteral reflux, which may be due to congenital abnormalities of bladder development, but there is no fistulous tract. A vitelline duct remnant may account for a Meckel diverticulum, or rarely a fistulous tract from small intestine to umbilicus.

29
Q

A 7-year-old boy is brought to the physician by his mother, who is concerned because he has become less active over the past 10 days. On physical examination, the boy has facial puffiness. Urinalysis shows no blood, glucose, or ketones, and microscopic examination shows no casts or crystals. The serum creatinine level is normal. A 24-hour urine collection yields 3.8 g of protein. He improves after corticosteroid therapy. He has two more episodes of proteinuria over the next few years, both of which respond to corticosteroid therapy. A renal biopsy is done. What is the most likely mechanism causing his disease?

□ (A) Immune complex–mediated glomerular injury
□ (B) Verocytotoxin-induced endothelial cell injury
□ (C) Cytotoxic T cell–mediated tubular epithelial cell injury
□ (D) Cytokine-mediated visceral epithelial cell injury
□ (E) IgA-mediated mesangial cell injury

A

(D) Steroid-responsive proteinuria in a child is typical of minimal change disease, in which the kidney looks normal by light microscopy, but there is foot process fusion by electron microscopy. The most likely cause of foot process fusion is a primary injury to visceral epithelial cells caused by T cell–derived cytokines. Immune complex deposition in membranous glomerulopathy can cause nephrotic syndrome, but is less common in children than adults and is not steroid-responsive. Certain verocytotoxin-producing Escherichia coli strains can cause hemolytic-uremic syndrome by injury to capillary endothelium. Acute cellular renal transplant rejection is mediated by T cell injury with tubulitis. IgA nephropathy with mesangial IgA deposition and consequent glomerular injury causes recurrent gross or microscopic hematuria and, far less commonly, nephrotic syndrome.

30
Q

A 49-year-old man saw his physician because he had increased swelling in the extremities for 2 months. Physical examination showed generalized edema. A 24-hour urine collection yielded 4.1 g of protein and albumin and globulins. Extensive testing did not indicate the presence of a systemic disease, such as diabetes mellitus or systemic lupus erythematosus. He did not respond to a course of corticosteroid therapy. A renal biopsy was done, and microscopic examination showed diffuse thickening of the basement membrane. Immunofluorescence staining with antibody to the C3 component of complement was positive in a granular pattern in the glomerular capillary loops. Two years later, he experiences increasing malaise. Laboratory studies now show serum creatinine level of 4.5 mg/dL and urea nitrogen level of 44 mg/dL. Which of the following immunological mechanisms was most likely responsible for the glomerular changes observed in the biopsy specimen?

□ (A) Antibodies that react with basement membrane collagen
□ (B) Antibodies against streptococci that cross-react with the basement membrane
□ (C) Release of cytokines by inflammatory cells
□ (D) Cytotoxic T cells directed against renal antigens
□ (E) Deposition of immune complexes on the basement membrane

A

(E) This patient has idiopathic membranous glomerulopathy, producing nephrotic syndrome. Diffuse basement membrane thickening, in the absence of proliferative changes, and granular deposits of IgG and C3 are typical of this condition. It is caused by the deposition of immune complexes on the basement membrane, which activates complement. Antibodies that react with basement membrane give rise to a linear immunofluorescence pattern, as in Goodpasture syndrome. Membranous glomerulopathy has no association with streptococcal infections. There also is no evidence of cytokine-mediated or T cell–mediated damage in this disease. In 85% of patients with membranous glomerulopathy, the cause of immune complex deposition is unknown. In the remaining 15%, an associated systemic disease (e.g., systemic lupus erythematosus) or some known cause of immune complex formation (e.g., drug reaction) exists.

31
Q

A 31-year-old woman experiences abdominal pain and sees her physician 1 week later after noticing blood in her urine. She has had three episodes of urinary tract infection during the past year. There are no remarkable findings on physical examination. Urinalysis shows 2+ hematuria; 1+ proteinuria; hypercalciuria; and no glucose or ketones. Microscopic examination of the urine shows numerous RBCs and oxalate crystals. An intravenous pyelogram shows linear striations radiating into the renal papillae, along with small cystic collections of contrast material in dilated collecting ducts. She is advised to increase her daily intake of fluids, and her condition improves. Which of the following conditions is most likely to be associated with these findings?

□ (A) Autosomal-dominant polycystic kidney disease
□ (B) Gout
□ (C) Medullary sponge kidney
□ (D) Multicystic renal dysplasia
□ (E) Autosomal-recessive polycystic kidney disease
□ (F) Urothelial carcinoma
□ (G) Vesicoureteral reflux

A

(C) The congenital disorder known as medullary sponge kidney (MSK) is present to some degree in 1% of adults. In MSK, cystic dilation of 1 to 5 mm is present in the inner medullary and papillary collecting ducts. MSK is bilateral in 70% of cases. Not all papillae are equally affected, although calculi are often present in dilated collecting ducts. Patients usually develop kidney stones, infection, or recurrent hematuria in the third or fourth decade. More than 50% of patients have stones. Autosomal-dominant polycystic kidney disease produces much larger cysts that involve the entire kidney, eventually leading to massive renomegaly. Uric acid crystals are present in gout and may be deposited in the medulla, but cysts do not form. Multicystic renal dysplasia may occur sporadically or as part of various genetic syndromes, such as Meckel-Gruber syndrome, in fetuses and newborns. Autosomal-recessive polycystic kidney disease is rare and leads to bilateral, symmetric renal enlargement manifested in utero, with renal failure evident at birth. Transitional cell carcinoma may be multifocal, but it produces masses, not cysts. Vesicoureteral reflux can lead to hydroureter, hydronephrosis, and an increased risk of infection.

32
Q

A 42-year-old man has had right flank pain for the past 2 days. On physical examination, his temperature is 37.4°C, pulse is 70/min, respirations are 14/min, and blood pressure is 130/85 mm Hg. Laboratory studies show a serum creatinine level of 1.1 mg/dL. Urinalysis shows no blood, protein, or glucose, and microscopic examination of the urine shows no WBCs or RBCs. Abdominal CT scan shows a 7-cm eccentric lesion of the upper pole of the right kidney. The lesion is well circumscribed and cystic with a thin wall and focal hemorrhage. What is the most likely diagnosis?

□ (A) Acute pyelonephritis 
□ (B) Acute tubular necrosis 
□ (C) Diabetic nephropathy
□ (D) Hydronephrosis
□ (E) Simple renal cyst
□ (F) Rapidly progressive glomerulonephritis 
□ (G) Renal cell carcinoma
□ (H) Urothelial carcinoma
A

(E) Simple cysts are common in adults, and multiple cysts may occur. The cysts are not as numerous as cysts occurring in autosomal-dominant polycystic kidney disease (ADPKD), and there is no evidence of renal failure. Simple cysts may be 10 cm, and hemorrhage sometimes occurs into a cyst. Multiple cysts sometimes develop in patients receiving long-term hemodialysis. Acute pyelonephritis is unlikely in this patient because of the absence of fever and WBCs in the urine. Acute pyelonephritis may be associated with small abscesses, but not with cysts, although in patients with ADPKD, cysts may become infected. Acute tubular necrosis follows ischemic or toxic injury, and there is evidence of renal failure. Diabetic nephropathy includes vascular and glomerular disease, but not cysts. Hydronephrosis may produce a focal obstruction of a calyx with dilation, but it does not produce an eccentric cyst. Glomerulonephritis is not associated with cyst formation. Neoplasms usually produce solid masses, although sometimes a renal cell carcinoma is cystic. The latter is much less common than a simple cyst, however.

33
Q

A 51-year-old woman has had recurrent urinary tract infections for the past 15 years. On many of these occasions, Proteus mirabilis was cultured from her urine. For the past 4 days, she has had a burning pain on urination and urinary frequency. On physical examination, her temperature is 37.9°C, pulse is 70/min, respirations are 15/min, and blood pressure is 135/85 mm Hg. There is marked tenderness on deep pressure over the right costovertebral angle and on deep abdominal palpation. Urinalysis shows a pH of 7.5; specific gravity 1.020; 1+ hematuria; and no protein, glucose, or ketones. Microscopic examination of the urine shows many RBCs, WBCs, and triple-phosphate crystals. Which of the following renal lesions is most likely to be present?

□ (A) Renal cell carcinoma 
□ (B) Acute tubular necrosis 
□ (C) Malignant nephrosclerosis 
□ (D) Staghorn calculus
□ (E) Papillary necrosis
A

(D) Recurrent urinary tract infections with urea-splitting organisms such as Proteus can lead to formation of magnesium ammonium phosphate stones. These stones are large, and they fill the dilated calyceal system. Because of their large size and projections into the calyces, such stones are sometimes called “staghorn calculi.” Infections are not a key feature of renal cell carcinoma. Cases of acute tubular necrosis typically occur from toxic or ischemic renal injuries. Malignant nephrosclerosis is primarily a vascular process that is not associated with infection. Papillary necrosis can complicate diabetes mellitus.

34
Q

A 53-year-old woman has had dysuria and urinary frequency for the past week. On physical examination, her temperature is 38°C, and she has pain on palpation over the left costovertebral angle. Laboratory findings show glucose, 177 mg/dL; hemoglobin A1c, 9.8%; hemoglobin, 13.1 g/dL; platelet count, 232,200/mm3; and WBC count, 11,320/mm3. Urinalysis shows a pH of 6.5; specific gravity 1.016; 2+ glucosuria; and no blood, protein, or ketones. Microscopic examination of the urine shows numerous neutrophils, and a urine culture is positive for Escherichia coli. Which of the following complications is most likely to develop in this patient?

□ (A) Acute tubular necrosis
□ (B) Necrotizing papillitis
□ (C) Crescentic glomerulonephritis 
□ (D) Hydronephrosis
□ (E) Renal calculi
A

(B) This patient has laboratory findings consistent with diabetes mellitus and clinical features of acute pyelonephritis caused by Escherichia coli infection. Necrotizing papillitis with papillary necrosis is a complication of acute pyelonephritis, and diabetic patients are particularly prone to this development. In the absence of diabetes mellitus, papillary necrosis develops when acute pyelonephritis occurs in combination with urinary tract obstruction. Papillary necrosis also can occur with long-term use of analgesics. Acute tubular necrosis typically occurs in acute renal failure caused by hypoxia (e.g., shock) or toxic injury (e.g., mercury). Crescentic glomerulonephritis causes rapidly progressive renal failure. Hydronephrosis occurs when urinary outflow is obstructed in the renal pelvis or in the ureter. Renal calculi can complicate conditions such as gout, but they do not complicate diabetes mellitus.

35
Q

A 42-year-old man has experienced increasing malaise for the past month. He is bothered by increasing swelling in the hands and legs. On physical examination, there is generalized edema. He is afebrile, and his blood pressure is 140/90 mm Hg. Urinalysis shows a pH of 6.5; specific gravity 1.017; 4+ proteinuria; and no blood, glucose, or ketones. Microscopic examination of the urine shows no casts or RBCs and 2 WBCs per high-power field. The 24-hour urine protein level is 4.2 g. A renal biopsy specimen is obtained, and immunofluorescence staining with antibody to the C3 component of complement produces the pattern shown in the figure. Which of the following underlying disease processes is most likely to be present?

□ (A) Chronic hepatitis B
□ (B) AIDS
□ (C) Multiple myeloma
□ (D) Recurrent urinary tract infection 
□ (E) Nephrolithiasis
A

(A) One of the most common causes of nephrotic syndrome in adults is membranous glomerulopathy, caused by immune complex deposition, shown here as granular deposits with C3. About 85% of cases are idiopathic, but some cases follow infections (e.g., hepatitis, malaria), or are associated with causes such as malignancies or autoimmune diseases. In some cases of AIDS, a nephropathy resembling focal segmental glomerulosclerosis occurs. Multiple myeloma can be complicated by systemic amyloidosis, which can involve the kidney. Recurrent urinary tract infection can cause chronic pyelonephritis. Nephrolithiasis may lead to interstitial nephritis, but it does not cause glomerular injury.

36
Q

A 58-year-old man is in stable condition after an acute myocardial infarction. Two days later, his urine output decreases, and the serum urea nitrogen level increases to 3.3 mg/dL. Oliguria persists for 5 days, followed by polyuria for 2 days. He is discharged from the hospital. Which of the following renal lesions best explains these renal abnormalities?

□ (A) Acute tubular necrosis
□ (B) Benign nephrosclerosis
□ (C) Acute renal infarction
□ (D) Hemolytic-uremic syndrome
□ (E) Rapidly progressive glomerulonephritis
A

(A) The most common cause of acute tubular necrosis is ischemic injury. The hypotension that develops after myocardial infarction causes decreased renal blood flow. Benign nephrosclerosis takes years to develop and is associated with benign essential hypertension. Emboli from mural thrombosis after myocardial infarction could reach the kidney, causing renal infarction, but these infarctions are small and focal. Hemolytic-uremic syndrome is a thrombotic microangiopathy that most often occurs in children after infection with enterotoxigenic Escherichia coli. A rapidly progressive glomerulonephritis does not follow ischemic injury and would not resolve as quickly as in this patient.

37
Q

Several members of a family developed chronic renal failure by age 50 years. Most are males. The affected individuals also developed visual problems. Some younger family members have proteinuria and hematuria on urinalysis. A renal biopsy specimen from a 20-year-old man shows prominent tubular foam cells and glomerular basement membrane thickening and thinning. Family members with this disease are most likely to have which of the following additional manifestations?

□ (A) Watery diarrhea
□ (B) Nerve deafness
□ (C) Presenile dementia 
□ (D) Dilated cardiomyopathy 
□ (E) Infertility
A

(B) These findings are characteristic of Alport syndrome, a form of hereditary nephritis. Most cases are inherited in an X-linked dominant pattern, but autosomal dominant and autosomal recessive patterns of inheritance also are seen. Most commonly, males are severely affected. Vision, hearing, and renal function are affected, but other organ systems are not affected

38
Q

A 65-year-old man recently retired after many years in a job that involved exposure to aniline dyes, including ß- naphthylamine. One month ago, he had an episode of hematuria that was not accompanied by abdominal pain. On physical examination, there are no abnormal findings. Urinalysis shows 4+ hematuria, and no ketones, glucose, or protein. Microscopic examination of the urine shows RBCs that are too numerous to count, 5 to 10 WBCs per high-power field, and no crystals or casts. The result of a urine culture is negative. What is the most likely diagnosis?

□ (A) Renal cell carcinoma
□ (B) Hemorrhagic cystitis
□ (C) Tubercular cystitis
□ (D) Urothelial carcinoma
□ (E) Squamous cell carcinoma of the urethra
A

(D) Exposure to arylamines markedly increases the risk of bladder cancer, which can occur decades after the initial exposure. After absorption, aromatic amines are hydroxylated into an active form, which is detoxified by conjugation with glucuronic acid and then excreted. Urinary glucuronidase splits the nontoxic conjugated form into the active carcinogenic form. Renal cell carcinomas also may manifest as painless hematuria, but exposure to aniline dyes is not a risk factor. Hemorrhagic cystitis occurs after radiation injury or treatment with cytotoxic drugs such as cyclophosphamide. Tubercular cystitis is typically a complication of renal tuberculosis. Squamous cell carcinoma is the most common malignancy of the urethra, but it is rare and has no relation to carcinogens.

39
Q

A 55-year-old woman has had poorly controlled hyperglycemia for many years. She sees her physician after experiencing burning pain on urination for 3 days. Physical examination shows a 2-cm ulceration on the skin of the heel and reduced sensation in the lower extremities. Her visual acuity is 20/100 bilaterally. Urinalysis shows 1+ proteinuria; 2+ glucosuria; and no blood, ketones, or urobilinogen. A urine culture contains more than 100,000 colony-forming units/mL of Klebsiella pneumoniae. Which of the following pathologic findings is most likely to be present in both kidneys?

□ (A) Deposits of IgG and C3 in the glomerular basement membrane
□ (B) Effacement of podocyte foot processes
□ (C) Glomerular crescents
□ (D) Mesangial deposits of IgA
□ (E) Necrotizing granulomatous vasculitis
□ (F) Nodular hyaline mesangial masses
□ (G) Thickening and thinning of the glomerular basement membrane

A

(F) This patient has diabetes mellitus. Nodular and diffuse glomerulosclerosis often occur in patients with long- standing diabetes mellitus. Infections with bacterial organisms also occur more frequently in patients with diabetes mellitus. Deposits of IgG and C3 in the glomerular basement membrane occur with forms of glomerulonephritis caused by immune complex deposition, including lupus nephritis and membranous glomerulonephritis. The only abnormality observed in minimal change disease is effacement of podocyte foot processes, but this change is not specific for minimal change disease and may be seen in other disorders that produce proteinuria. Crescentic glomerulonephritis is not typically seen in diabetes mellitus. IgA deposition in the mesangium occurs in IgA nephropathy (Berger disease). A necrotizing granulomatous vasculitis can be present in the kidneys of patients with Wegener’s granulomatosis. Thickening and thinning of glomerular basement membranes occurs in Alport syndrome.

40
Q

A 17-year-old girl has had arthralgias and myalgias for several months. During the past week, she has noticed a decreased output of urine, which is reddish brown. On physical examination, her blood pressure is 160/100 mm Hg, and she has an erythematous malar skin rash. The ANA and anti–double-stranded DNA test results are positive. The serum urea nitrogen level is 52 mg/dL. Which of the following urinalysis findings is most likely to be reported for this patient?

□ (A) Eosinophils
□ (B) Glucose
□ (C) Ketones
□ (D) Myoglobin
□ (E) Oval fat bodies
□ (F) RBC casts
□ (G) Triple phosphate crystals 
□ (H) Uric acid crystals
□ (I) Waxy casts
A

(F) This patient has findings of systemic lupus erythematosus, an autoimmune disease that often manifests with renal involvement. There are several forms of lupus nephritis, and they tend to produce a nephritic pattern of involvement. Because these patients have leakage of RBCs from damaged glomeruli and proteinuria, RBC casts are found in the urine. Eosinophils may appear in the urine as a result of drug-induced interstitial nephritis. Glucosuria and ketonuria are features of type 1 diabetes mellitus. Myoglobinuria results most often from rhabdomyolysis, which can occur after severe crush injuries. Oval fat bodies are sloughed tubular cells containing abundant lipid that are characteristic of nephrotic syndromes. Triple phosphate crystals are typical findings in patients with infections caused by urease-positive bacteria, such as Proteus. Uric acid crystals form in acidic urine when uricosuria is present; this is a characteristic feature in some patients with gout. Waxy casts form in dilated, damaged tubules.

41
Q

A 28-year-old woman has had dysuria, frequency, and urgency for the past 2 days. On physical examination, her temperature is 37.6°C. A urine culture grows greater than 100,000 colonies/mL of Escherichia coli. She is treated with antibiotic therapy. If the problem continues to recur, the patient is likely to be at greatest risk for development of which of the following renal diseases?

□ (A) Diffuse glomerulosclerosis 
□ (B) Chronic glomerulonephritis
□ (C) Amyloidosis
□ (D) Membranous glomerulonephritis 
□ (E) Chronic pyelonephritis
A

(E) Most cases of pyelonephritis result from ascending bacterial infections, which are more common in women. Recurrent urinary tract infections complicated by vesicoureteral reflux cause progressive interstitial damage and scarring, which can lead to chronic pyelonephritis with renal failure. Diffuse glomerulosclerosis is a feature of diabetes mellitus. Glomerular injury is not the major consequence of renal infections. Some cases of membranous glomerulonephritis are preceded by chronic infections, such as hepatitis B or malaria, but recurrent urinary tract infections alone are unlikely antecedents. Some chronic infections (e.g., lung abscess, pulmonary tuberculosis) can lead to reactive systemic amyloidosis that may involve the kidney. Recurrent urinary tract infections do not cause amyloidosis, however.

42
Q

A sexually active, 26-year-old man has had pain on urination for the past 4 days. On physical examination, there are no lesions on the penis. He is afebrile. Urinalysis shows no blood, ketones, protein, or glucose. Microscopic examination of the urine shows few WBCs and no casts or crystals. What infectious agent is most likely to produce these findings?

□ (A) Chlamydia trachomatis 
□ (B) Mycobacterium tuberculosis 
□ (C) Herpes simplex virus
□ (D) Candida albicans
□ (E) Treponema pallidum
A

(A) This patient has urethritis. The most common cause of nongonococcal urethritis in men is Chlamydia trachomatis. The condition is a nuisance; however, the behavior that led to the infection can place the patient at risk of other sexually transmitted diseases. Tuberculosis of the urinary tract is uncommon. Herpes simplex can produce painful vesicles on the skin. Candida infections typically occur in immunocompromised patients or in patients receiving long-term antibiotic therapy. A syphilitic chancre on the penis is an indicator of Treponema pallidum infection.

43
Q

A 61-year-old woman sees the physician because she has experienced increasing malaise for the past 5 years. On physical examination, there are no abnormalities other than a blood pressure of 150/95 mm Hg. One week later, she dies suddenly. At autopsy, both kidneys have the external (left panel) and bisected (right panel) appearance as shown in the figure. Which of the following conditions was the most probable cause of death?

□ (A) Metastatic Wilms tumor
□ (B) Ruptured berry aneurysm
□ (C) Acute tubular necrosis
□ (D) Disseminated intravascular coagulation 
□ (E) Pneumothorax
A

(B) These findings are characteristic of autosomal-dominant polycystic kidney disease (ADPKD). As seen in the figure, several large cysts have completely replaced the kidney. In autosomal-recessive polycystic kidney disease, which typically manifests in fetal and neonatal life, the kidneys have a smooth external appearance. On cut section, many small cysts give the kidney a spongelike appearance. About 10% to 30% of affected patients with ADPKD have an intracranial berry aneurysm, and some of these can rupture without warning. Wilms tumor does not arise in a polycystic kidney. Acute tubular necrosis is the result of ischemic or toxic renal injuries. Disseminated intravascular coagulation may complicate hemolytic-uremic syndrome. Pulmonary disease does not accompany adult polycystic kidney disease.

44
Q

For the past 20 years, a 69-year-old man with chronic arthritis has taken more than 3 g of analgesics per day, including phenacetin, aspirin, and acetaminophen. He sees his physician because of increasing malaise, nausea, and diminished mentation. On physical examination, his blood pressure is 156/92 mm Hg. Laboratory findings show serum urea nitrogen level of 68 mg/dL and creatinine level of 7.1 mg/dL. CBC shows hemoglobin, 11.7 g/dL; hematocrit, 35.1%; platelet count, 188,500/mm3; and WBC count, 5385/mm3. Which of the following renal diseases is this patient most likely to develop?

□ (A) Hydronephrosis
□ (B) Chronic glomerulonephritis 
□ (C) Renal papillary necrosis 
□ (D) Renal cell carcinoma 
□ (E) Acute tubular necrosis
A

(C) This patient has analgesic nephropathy, which damages the renal interstitium and can give rise to papillary necrosis. Hydronephrosis is unlikely to develop because there is no urinary tract obstruction in analgesic nephropathy. There is an increased risk of transitional cell carcinoma of the renal pelvis, however. The toxic injury that occurs with analgesic use is slowly progressive and not acute, in contrast to the course of acute tubular necrosis.

45
Q

A 58-year-old woman sees her physician for a routine health maintenance examination. The only abnormality on physical examination is a blood pressure of 168/109 mm Hg. Urinalysis shows a pH of 7.0; specific gravity 1.020; 1+ proteinuria; and no blood, glucose, or ketones. An abdominal ultrasound scan shows bilaterally and symmetrically small kidneys with no masses. The ANA test result is negative. The serum urea nitrogen level is 51 mg/dL, and the creatinine level is 4.7 mg/dL. The hemoglobin A1c concentration is within the reference range. What is the most likely diagnosis?

□ (A) Lupus nephritis
□ (B) Autosomal-dominant polycystic kidney disease 
□ (C) Chronic glomerulonephritis
□ (D) Nodular glomerulosclerosis
□ (E) Amyloidosis
A

(C) Chronic glomerulonephritis may follow specific forms of acute glomerulonephritis. In many cases, however, it develops insidiously with no known cause. With progressive glomerular injury and sclerosis, both kidneys become smaller, and their surfaces become granular. Hypertension often develops because of renal ischemia. Regardless of the initiating cause, these “end-stage” kidneys appear morphologically identical. They have sclerotic glomeruli, thickened arteries, and chronic inflammation of interstitium. Because the patient’s ANA test result is negative, lupus is unlikely. Polycystic kidney disease and amyloidosis would cause the kidney size to increase, not decrease. The normal hemoglobin A1c concentration indicates that the patient does not have diabetes mellitus. Nodular glomerulosclerosis is typical of diabetes mellitus with an elevated hemoglobin A1c.

46
Q

A previously healthy, 21-year-old man sees his physician because he notices blood in his urine. He reports no dysuria, frequency, or hesitancy of urination. On physical examination, there are no abnormal findings. Laboratory findings show a serum urea nitrogen level of 39 mg/dL and creatinine level of 4.1 mg/dL. A renal biopsy specimen is obtained; the immunofluorescence pattern of staining with antibody against human IgG is shown in the figure. Which of the following serum laboratory studies is most likely to be positive in this patient?

□ (A) Antistreptolysin O antibody
□ (B) HIV antibody
□ (C) Anti–glomerular basement membrane antibody 
□ (D) Hepatitis B surface antibody
□ (E) C3 nephritic factor
A

(C) The linear pattern of staining shown in the figure indicates the presence of anti–glomerular basement membrane antibodies. Such antibodies are typically seen in Goodpasture’s syndrome. The antistreptolysin O titer is increased in poststreptococcal glomerulonephritis, which typically has a granular pattern of immune complex deposition. HIV infection can lead to a nephropathy that resembles focal segmental glomerulosclerosis, in which IgM and C3 are deposited in the mesangial areas of affected glomeruli. Some cases of membranous glomerulonephritis are associated with hepatitis B virus infection, but the immune complex deposition is granular, not linear. The C3 nephritic factor can be a marker for type II membranoproliferative glomerulonephritis.

47
Q

A 33-year-old woman has had fever and increasing fatigue for the past 2 months. Over the past year, she has noticed soreness of her muscles and joints and has had a 4-kg weight loss. On physical examination, her temperature is 37.5°C, pulse is 80/min, respirations are 15/min, and blood pressure is 145/95 mm Hg. She has pain on deep inspiration, and a friction rub is heard on auscultation of the chest. Laboratory findings show glucose, 73 mg/dL; total cholesterol, 160 mg/dL; total protein, 5.2 g/dL; albumin, 2.9 g/dL; total bilirubin, 0.9 mg/dL; and creatinine, 2.4 mg/dL. Serum complement levels are decreased. CBC shows hemoglobin of 9.7 g/dL, platelet count of 85,000/mm3, and WBC count of 3560/mm3. A renal biopsy specimen shows a diffuse proliferative glomerulonephritis with extensive granular immune deposits of IgG and C1q in capillary loops and mesangium. After being treated with immunosuppressive therapy consisting of prednisone and cyclophosphamide, her condition improves. Which of the following serologic studies is most likely to be positive in this patient?

□ (A) Anticentromere antibody
□ (B) Anti–double-stranded DNA antibody
□ (C) Anti–DNA topoisomerase I antibody
□ (D) Anti–glomerular basement membrane antibody 
□ (E) Antihistone antibody
□ (F) ANCA
□ (G) Antiribonucleoprotein
A

(B) This patient has findings characteristic of systemic lupus erythematosus (SLE) with lupus nephritis. Systemic problems include fever, arthralgias, myalgias, pancytopenia, and serositis with pericarditis and pleuritis. Renal disease is common in SLE, and a renal biopsy helps to determine the severity of involvement and the appropriate therapy. Anticentromere antibody is most specific for limited scleroderma (CREST syndrome), which is unlikely to have renal involvement. Anti–DNA topoisomerase I antibody is more specific for diffuse scleroderma, which does have renal involvement, although usually this manifests as vascular disease and not as glomerulonephritis. Anti–glomerular basement membrane antibody is characteristic of Goodpasture’s syndrome, in which IgG antibody is deposited in a linear fashion along glomerular capillary basement membranes. Antihistone antibody may be present in drug-induced lupus. ANCAs can be seen in some forms of vasculitis, such as Wegener’s granulomatosis or microscopic polyangiitis. Antiribonucleotide protein is present in mixed connective tissue disease, which has some features of SLE, but usually does not include severe renal involvement.

48
Q

A 66-year-old woman died of an acute myocardial infarction. At autopsy, both kidneys were decreased in size (about 120 g each) with a finely granular cortical surface. The representative appearance of the kidney under high magnification is shown in the figure. Which of the following clinical abnormalities most likely accompanied this lesion?

□ (A) Oliguria
□ (B) Benign hypertension 
□ (C) Malignant hypertension 
□ (D) Hematuria
□ (E) Flank pain
A

(B) The figure shows hyaline arteriolosclerosis, which typically occurs in patients with benign hypertension. Similar changes can be seen with aging in the absence of hypertension. Oliguria is a sign of acute renal failure that does not complicate benign essential hypertension, a slowly progressive disease that is often clinically silent. Blood pressure screening is an important method that can identify patients with hypertension before significant organ damage has occurred. Malignant hypertension causes distinctive renal vascular lesions that include fibrinoid necrosis and hyperplastic arteriosclerosis. Hematuria may be present in malignant hypertension from vascular injury, but it is not a feature of benign hypertension. Flank pain is a symptom of acute pyelonephritis and some renal neoplasms.

49
Q

A 79-year-old man has had increasing back pain and fatigue for the past 6 months. On physical examination, there are no remarkable findings. Laboratory studies include a CBC with hemoglobin of 9.6 g/dL, platelet count of 241,600/mm3, and WBC count of 7160/mm3. The serum total protein is 9.8 g/dL, albumin is 3.6 g/dL, glucose is 72 mg/dL, creatinine is 3.3 mg/dL, and urea nitrogen is 30 mg/dL. A dipstick urinalysis shows a pH of 7; specific gravity 1.011; and no blood, protein, or glucose. One month later, he develops a cough with fever, and Streptococcus pneumoniae is cultured from his sputum. Despite antibiotic therapy, he develops sepsis and dies. At autopsy, the kidneys are normal in size, but microscopic examination shows dilated tubules filled with amorphous blue-to-pink casts and occasional multinucleated giant cells. What is the most likely underlying cause of this patient’s death?

□ (A) Cystinuria
□ (B) Diabetes mellitus
□ (C) Gout
□ (D) Multiple myeloma
□ (E) Parathyroid adenoma
□ (F) Systemic lupus erythematosus
A

(D) This patient has a high serum globulin level from the presence of a monoclonal protein, and the back pain is probably caused by lytic lesions in the spine. Patients with myeloma often have Bence Jones proteinuria (not detected by the standard dipstick urinalysis), and some have cast nephropathy (as in this case), which can cause acute or, more commonly, chronic renal failure (as in this case). Cystinuria is an uncommon condition arising from defective transport of the amino acids cystine, lysine, arginine, and ornithine by the brush borders of renal tubule and intestinal epithelial cells. Excessive amounts of these amino acids are lost in the urine, leading to stone formation (the distinctive crystals look like miniature “stop” signs). Diabetic nephropathy can take many forms, but cast nephropathy is not one of them. Gouty deposits in the kidney are not in the form of casts, and uric acid crystals form at acidic pH. Hypercalcemia from a parathyroid adenoma can increase urine calcium excretion, favoring formation of stones, but not casts. Systemic lupus erythematosus is more likely to cause glomerulonephritis.

50
Q

A 30-year-old woman with a history of recurrent urinary tract infections has had a high fever for the past 3 days. On physical examination, her temperature is 38.4°C. There is marked abdominal tenderness on deep palpation. A renal ultrasound scan shows an enlarged right kidney with pelvic and calyceal enlargement and cortical thinning; the left kidney appears normal. A right nephrectomy is done, and microscopic examination shows inflammatory infiltrates extending from the medulla to the cortex, with tubular destruction and extensive interstitial fibrosis. Lymphocytes, plasma cells, and neutrophils are abundant. Which of the following is most likely to produce these findings?

□ (A) Benign nephrosclerosis
□ (B) Vesicoureteral reflux
□ (C) Lupus nephritis
□ (D) Systemic amyloidosis
□ (E) Congestive heart failure
□ (F) Autosomal-dominant polycystic kidney disease
A

(B) These changes are characteristic of chronic pyelonephritis. Urinary tract obstruction favors recurrent urinary tract infection. Vesicoureteral reflux propels infected urine from the urinary bladder to the ureters and renal pelvis and predisposes to infection. Benign nephrosclerosis is a vascular disease that does not carry a risk of infection. Lupus nephritis is associated with extensive inflammatory changes of glomeruli that are noninfectious. Amyloidosis can lead to progressive renal failure as a result of amyloid deposition in the glomeruli; however, amyloid does not evoke an inflammatory response. Congestive heart failure may predispose to acute tubular necrosis. Autosomal-dominant polycystic kidney disease is a bilateral process; patients usually are not symptomatic until middle age.

51
Q

A 32-year-old man with a history of intravenous drug use comes to the emergency department because he has had a high fever for the past 2 days. On physical examination, his temperature is 38.4°C. He has a palpable spleen tip, bilateral costovertebral angle tenderness, and diastolic cardiac murmur. Laboratory findings show a serum urea nitrogen level of 15 mg/dL. Urinalysis shows 2+ hematuria, and no glucose, protein, or ketones. A blood culture is positive for Staphylococcus aureus. Which of the following best describes the kidneys in this patient?

□ (A) Bilaterally enlarged kidneys replaced by 1- to 4-cm, fluid-filled cysts
□ (B) Bilaterally shrunken kidneys with uniformly finely granular cortical surfaces
□ (C) Irregular cortical scars in asymmetrically shrunken kidneys with marked calyceal dilation
□ (D) Marked bilateral renal pelvic and calyceal dilation with thinning of the cortices
□ (E) Normal-sized kidneys with smooth cortical surfaces
□ (F) Scattered petechial hemorrhages in slightly swollen kidneys
□ (G) Wedge-shaped regions of yellow-white cortical necrosis involving both kidneys

A

(G) This patient is septic, and the heart murmur strongly suggests infective endocarditis. Cardiac lesions are the source of emboli (from valvular vegetations or mural thrombi) that can lodge in renal artery branches, producing areas of coagulative necrosis. These areas of acute infarction typically are wedge-shaped on cut section because of the vascular flow pattern. Bilaterally enlarged, cystic kidneys are typical of autosomal-dominant polycystic kidney disease. Small, shrunken kidneys represent an end stage of many chronic renal diseases. Irregular cortical scars with pelvicaliceal dilation may represent hydronephrosis complicated by infection in chronic pyelonephritis, whereas dilation alone points to obstructive uropathy, such as occurs with nodular prostatic hyperplasia. This patient’s kidneys may have been normal- sized and smooth-surfaced before this event. Petechiae and edema may be seen in hyperplastic arteriolosclerosis associated with malignant hypertension.

52
Q
Three years ago, a 47-year-old woman had a mastectomy of the right breast to remove an infiltrating ductal carcinoma. She now has bone pain, and a radionuclide scan shows multiple areas of increased uptake in the vertebrae, ribs, pelvis, and right femur. Urinalysis shows a specific gravity of 1.010, which remains unchanged after water deprivation for 12 hours. She undergoes several courses of chemotherapy over the next year. During this time, the serum urea nitrogen level progressively increases. Which of the following abnormal laboratory findings is most likely to be reported for this patient?
□ (A) Hepatitis B surface antigenemia 
□ (B) Hypercalcemia
□ (C) Hypercholesterolemia
□ (D) Hypergammaglobulinemia 
□ (E) Hyperglycemia
□ (F) Hyperuricemia
A

(B) This patient has findings characteristic of nephrocalcinosis resulting from hypercalcemia. One of the most common causes of hypercalcemia in adults is metastatic disease. The hypercalcemia produces a chronic tubulointerstitial disease of the kidneys that is initially manifested by loss of concentrating ability. With continued hypercalcemia, there is progressive loss of renal function. Urinary tract stones formed of calcium oxalate also may be present. Some patients with membranous glomerulonephritis have a positive serologic test result for hepatitis B surface antigen. Hypercholesterolemia may be seen in some cases of minimal change disease, which can occur in Hodgkin lymphoma and other lymphoproliferative malignancies. Hypergammaglobulinemia with a monoclonal protein (M protein) may be present in multiple myeloma, but not in breast cancer. Hyperglycemia can occur in diabetes mellitus, but patients with cancer are not at increased risk of developing diabetes mellitus. Hyperuricemia occurs in some cases of gout. It also can occur in patients with neoplasms (particularly lymphomas and leukemias) that have a high proliferation rate and are treated with chemotherapy. In these cases, extensive cell death (lysis syndrome) causes acute elevations in uric acid levels, leading to urate nephropathy.

53
Q

A 45-year-old man has experienced increasing malaise, nausea, and reduced urine output for the past 3 days. On physical examination, he is afebrile and normotensive. Laboratory findings show a serum creatinine level of 2.5 mg/dL. Urinalysis shows hematuria, but no pyuria or glucosuria. A renal biopsy is done; the immunofluorescence pattern with antifibrinogen is shown in the figure. Which of the following additional studies is most useful for classification and treatment of this disease?

□ (A) ANA titer
□ (B) Anti–glomerular basement membrane antibody test 
□ (C) HIV titer
□ (D) Quantitative serum immunoglobulins
□ (E) Rheumatoid factor
□ (F) Urine immunoelectrophoresis
A

(B) The renal biopsy specimen shows glomerular crescents, which indicate rapidly progressive glomerulonephritis. Crescentic glomerulonephritis is divided into three groups on the basis of immunofluorescence: type I (anti–glomerular basement membrane [GBM] disease), type II (immune complex disease), and type III (characterized by absence of anti- GBM antibodies or immune complexes). Each type has a different cause and treatment. The presence of anti-GBM antibodies suggests Goodpasture’s syndrome; patients with this disorder require plasmapheresis. Type II crescentic glomerulonephritis can occur in systemic lupus erythematosus, in Henoch-Schönlein purpura, and after infections. Causes of type III crescentic glomerulonephritis include Wegener’s granulomatosis and microscopic polyangiitis. Immunofluorescence studies are crucial for the classification and treatment of crescentic glomerulonephritis. A positive ANA test result may be reported in patients with lupus nephritis, which uncommonly manifests with glomerular crescents. HIV nephropathy has features similar to those of focal segmental glomerulosclerosis, which is not rapidly progressive. Quantitative serum immunoglobulins are not helpful because the important consideration is the pattern of immune deposits in the kidney. Rheumatoid factor is present in rheumatoid arthritis, which is not associated with renal complications. Urine immunoelectrophoresis is useful in categorizing a monoclonal gammopathy.

54
Q

A 28-year-old man is diagnosed with acute myelogenous leukemia (M2). After induction with a multiagent chemotherapy protocol, he has an episode of lower abdominal pain accompanied by passage of red-colored urine. He has no fever, dysuria, or urinary frequency. On physical examination, there are no remarkable findings. Urinalysis shows a pH of 5.5; specific gravity 1.021; 2+ hematuria; and no protein, ketones, or glucose. There are no remarkable findings on an abdominal radiograph. Which of the following additional urinalysis findings is most likely to be reported for this patient?

 □ (A) Bence Jones protein
□ (B) Eosinophils
□ (C) Myoglobin
□ (D) Oval fat bodies
□ (E) RBC casts
□ (F) Triple phosphate crystals 
□ (G) Uric acid crystals
□ (H) Waxy casts
□ (I) WBC casts
A

(G) The rapid cell turnover in acute leukemias and cell death from treatment cause the release of purines from the cellular DNA breakdown. The resulting hyperuricemia can predispose to the formation of uric acid stones. Renal stones can produce colicky pain when they pass down the ureter and through the urethra, and the local trauma to the urothelium can produce hematuria. Uric acid stones form in acidic urine. In contrast to stones containing calcium, uric acid stones are radiolucent and are not visible on a plain radiograph. The urine dipstick is sensitive to albumin, but not to globulins; a separate test for Bence Jones protein may be positive, although the dipstick protein result is negative. Bence Jones proteinuria is characteristic of multiple myeloma, however, not of leukemias or lymphomas. Eosinophils may appear in the urine in drug-induced interstitial nephritis. Myoglobin can cause the dipstick reagent for blood to become positive in the absence of RBCs or hemoglobin. Myoglobinuria results most often from rhabdomyolysis, which can occur after severe crush injuries. Oval fat bodies are sloughed tubular cells containing abundant lipid; they are characteristic of nephrotic syndromes. RBC casts appear in nephritic syndromes as a result of glomerular injury. Triple phosphate crystals are typical of infections with urease-positive bacteria, such as Proteus. Waxy casts form in dilated, damaged tubules. WBC casts alone are most indicative of acute pyelonephritis, but they can appear in conjunction with other cellular elements in severe glomerular injury.

55
Q

A 44-year-old man has developed a fever, nonproductive cough, and decreased urine output over the past 3 days. On physical examination, his temperature is 37.7°C, and blood pressure is 145/95 mm Hg. He has sinusitis. On auscultation, crackles are heard over all lung fields. A chest radiograph shows bilateral patchy infiltrates and nodules. The serum creatinine level is 4.1 mg/dL, and the urea nitrogen level is 43 mg/dL. The results of serologic testing are negative for ANA, but positive for C-ANCA. A renal biopsy specimen shows glomerular crescents and granulomatous vasculitis. The result of immunofluorescence staining with anti-IgG and anti-C3 antibodies is negative. What is the most likely diagnosis?

□ (A) Focal segmental glomerulosclerosis
□ (B) Goodpasture syndrome
□ (C) Lupus nephritis
□ (D) Membranoproliferative glomerulonephritis type II 
□ (E) Membranous glomerulonephritis
□ (F) Postinfectious glomerulonephritis
□ (G) Wegener's granulomatosis
A

(G) Wegener granulomatosis causes rapidly progressive glomerulonephritis characterized by epithelial crescents in Bowman space. Several features differentiate Wegener granulomatosis from other forms of crescentic glomerulonephritis (e.g., Goodpasture syndrome), including the presence of granulomatous vasculitis, the absence of immune complexes or anti–glomerular basement membrane (GBM) antibodies, and the presence of C-ANCA. Focal segmental glomerulosclerosis does not affect renal vessels and is unlikely to produce crescents with a rapidly progressive presentation. Goodpasture syndrome is a form of rapidly progressive glomerulonephritis with crescent formation, but a granulomatous vasculitis is not present, and there is anti-GBM antibody, not C-ANCA. Lupus nephritis, membranoproliferative glomerulonephritis, and postinfectious glomerulonephritis occasionally can have a rapidly progressive course with crescent formation, but they do not produce granulomatous vasculitis. In patients with lupus, the ANA test result is often positive. Membranous glomerulonephritis is most likely to produce nephrotic syndrome without crescents.

56
Q

A 65-year-old woman has recently experienced several transient ischemic attacks. On physical examination, the only abnormal finding is a blood pressure of 150/95 mm Hg. Urinalysis shows 1+ proteinuria, and no glucose, blood, or ketones. Microscopic examination of the urine shows no RBCs or WBCs and few oxalate crystals. On abdominal ultrasound, the kidneys are slightly decreased in size. Which of the following renal lesions is most likely to be present in this patient?

□ (A) Crescentic glomerulonephritis 
□ (B) Hyaline arteriolosclerosis 
□ (C) Mesangial cell proliferation 
□ (D) Arteriolar fibrinoid necrosis 
□ (E) Acute tubular necrosis
□ (F) Interstitial nephritis
A

(B) Hyaline arteriosclerosis, characterized by thickening and hyalinization of small arteries and arterioles, is typically seen in patients with long-standing benign hypertension. Such a change also occurs with aging. Vascular narrowing causes ischemic changes that are slow and progressive. There is diffuse scarring and shrinkage of the kidneys. RBCs and RBC casts are a feature of crescentic glomerulonephritis, which typically is a rapidly progressive form of renal failure. Mesangial proliferation is a feature of some forms of glomerulonephritis. Fibrinoid necrosis in arterioles is seen in malignant hypertension. Acute tubular necrosis results from anoxic or toxic injury to the renal tubules. In interstitial nephritis, more cells would be seen in the urine sediment.

57
Q

Over the past 72 hours, a 44-year-old man has experienced worsening headache, nausea, and vomiting. On physical examination, his blood pressure is 276/158 mm Hg, and there is bilateral papilledema. Urinalysis shows 2+ proteinuria; 1+ hematuria; and no glucose or ketones. Which of the following renal lesions is most likely to be present in this patient?

□ (A) Papillary necrosis 
□ (B) Acute infarction 
□ (C) Necrotizing arteriolitis
□ (D) Acute tubular necrosis
□ (E) Acute pyelonephritis
A

(C) This patient has malignant hypertension. Necrotizing arteriolitis and hyperplastic arteriolosclerosis are the distinctive vascular lesions of malignant hypertension. Papillary necrosis is more likely to complicate diabetic nephropathy or analgesic nephropathy. Infarction of the kidney may result from emboli originating in the systemic circulation. Malignant hypertension does not damage the large systemic vessels, however. Acute tubular necrosis is seen in hypoxic or toxic injury to the renal tubules. Acute pyelonephritis is a febrile illness, without severe blood pressure elevation.

58
Q

A 35-year-old, previously healthy man is found dead in his home. At autopsy, the medical examiner notices bilaterally enlarged kidneys that contain multiple, irregularly arranged cysts of different shapes and sizes. There is a 0.5-cm nonruptured intracerebral berry aneurysm of the anterior communicating artery. There are scattered 1- to 2-cm, fluid-filled liver cysts involving 10% of the parenchymal volume. Postmortem laboratory testing of the urine and blood shows markedly elevated levels of cocaine and its metabolite, benzoylecgonine. Which of the following is the most appropriate conclusion to be drawn from these findings?

□ (A) He had lesions related to chronic use of cocaine
□ (B) He had autosomal-recessive polycystic kidney disease, but survived to adulthood
□ (C) His surviving family (children, siblings, and parents) should be evaluated for a similar condition
□ (D) The immediate cause of death is berry aneurysm
□ (E) The underlying cause of death is autosomal-dominant polycystic kidney disease

A

(C) The combination of cysts in the kidney and berry aneurysms in the brain is characteristic of adult autosomal- dominant polycystic kidney disease (ADPKD). The cysts also may appear in the liver and pancreas. Because of the autosomal dominant pattern of inheritance, with high penetrance of the gene, first-degree relatives are at risk of having the same disorder, and should be evaluated by ultrasound or other imaging techniques. This evaluation is particularly important because many patients remain asymptomatic until the onset of renal failure as adults. Cocaine use can increase the risk of hemorrhages, but no hemorrhage was found in this case. The cause of death was cocaine intoxication. Autosomal-recessive polycystic kidney disease is unlikely to remain asymptomatic from birth. Because the berry aneurysm had not ruptured, it was not the cause of death. About 1% of all individuals have a berry aneurysm, whereas at least 10% of individuals with ADPKD have such an aneurysm. Most berry aneurysms do not rupture, however. The patient did not die as a consequence of ADPKD; had he lived, he might have developed complications from renal failure, leading to death decades later.

59
Q

A 12-year-old girl has experienced increasing malaise for the past 2 weeks. On physical examination, she has periorbital edema. The child is afebrile. Laboratory findings show proteinuria on dipstick urinalysis, but no hematuria or glucosuria. Microscopic examination of the urine shows numerous oval fat bodies. The serum creatinine level is 2.3 mg/dL. She receives a course of corticosteroid therapy, but does not improve. A renal biopsy is done; the biopsy specimen shows that approximately 50% of the glomeruli in the specimen are affected by the lesion shown in the figure. What is the most likely diagnosis?

□ (A) Focal segmental glomerulosclerosis
□ (B) Lipoid nephrosis
□ (C) Membranoproliferative glomerulonephritis type I
□ (D) Membranoproliferative glomerulonephritis type II
□ (E) Nodular glomerulosclerosis
□ (F) Postinfectious glomerulonephritis
□ (G) Rapidly progressive glomerulonephritis

A

(A) The biopsy specimen shows sclerosis of only a segment of the glomerulus (segmental lesion), and because only 50% of the glomeruli are affected, this is focal disease. Focal segmental glomerulosclerosis manifests clinically with nephrotic syndrome that does not respond to corticosteroid therapy. In contrast, corticosteroid-responsive nephrotic syndrome in children is typically caused by minimal change disease (lipoid nephrosis); this disease is not associated with any glomerular change seen under the light microscope. Type I or type II membranoproliferative glomerulonephritis is more likely to produce a nephritic syndrome in adults. A diabetic patient with nephrotic syndrome is likely to have nodular glomerulosclerosis or diffuse thickening of the basement membrane. Sore throat (pharyngitis) followed by nephritic syndrome is the typical clinical history of acute proliferative poststreptococcal (postinfectious) glomerulonephritis. A rapidly progressive glomerulonephritis is associated with hematuria, and glomerular crescents are present.

60
Q

A 19-year-old woman has had a fever and chills accompanied by right flank pain for the past 3 days. On physical examination, her temperature is 38.3°C, her blood pressure is 150/90 mm Hg, and there is right costovertebral angle tenderness. Laboratory findings show a serum glucose level of 77 mg/dL and creatinine level of 1 mg/dL. Urinalysis shows a pH of 6.5; specific gravity 1.018; and no protein, blood, glucose, or ketones. Microscopic examination of the urine shows many WBCs and WBC casts. Which of the following factors is most important in the pathogenesis of the renal disease affecting this patient?

□ (A) Age 
□ (B) Sex
□ (C) Vesicoureteral reflux
□ (D) Blood pressure
□ (E) Focus of infection in the lungs
A

(C) This patient has acute pyelonephritis. Acquired or congenital vesicoureteral reflux is extremely important in the pathogenesis of ascending urinary tract infections because it allows bacteria to ascend from the urinary bladder into the ureter and the pelvis. Urinary tract infections generally are more common in females because of their shorter urethra, but in the absence of vesicoureteral reflux, the infections tend to remain localized in the urinary bladder. Older women and sexually active women are at increased risk of urinary tract infections. Hypertension can cause renal vascular narrowing and ultimately impair renal function, but it does not predispose to infections. Foci of infection in the lungs can seed the kidney hematogenously, but this route is far less common than ascending infection.

61
Q

Several days after eating a hamburger, chili, and ice cream at a home barbecue, a 5-year-old girl develops cramping abdominal pain and diarrhea. The next day, she has decreased urine output. On physical examination, there are petechial hemorrhages on the skin. Her temperature is 37°C, pulse is 90/min, respirations are 18/min, and blood pressure is 90/50 mm Hg. A stool sample is positive for occult blood. Laboratory findings show hemoglobin, 10.8 g/dL; hematocrit, 32.4%; platelet count, 64,300/mm3; and WBC count, 6480/mm3. The peripheral blood smear shows schistocytes, and the serum D-dimer level is elevated. Which of the following is the most likely causative organism?

□ (A) Candida albicans 
□ (B) Proteus mirabilis 
□ (C) Clostridium difficile 
□ (D) Escherichia coli
□ (E) Staphylococcus aureus
A

(D) This girl has hemolytic-uremic syndrome. Some strains of Escherichia coli, which can contaminate ground beef products, may elaborate a toxin that damages endothelium, causing this syndrome. Hemolytic-uremic syndrome most often occurs in children and is one of the most common causes of acute renal failure in children. Candidal urinary tract infections typically affect the urinary bladder. Proteus is a common cause of bacterial urinary tract infections. Clostridium difficile is best known for causing a pseudomembranous enterocolitis, not renal lesions. Staphylococcus aureus can cause urinary tract infections.

62
Q

A 17-year-old boy is involved in a motor vehicle accident in which he sustains severe blunt trauma to the extremities and abdomen. Over the next 3 days, he develops oliguria and dark brown urine. The urine dipstick analysis is positive for myoglobin and for blood, but microscopic examination of the urine shows no RBCs. His serum urea nitrogen level increases to 38 mg/dL, and he undergoes hemodialysis for 3 weeks. His condition improves, but the urine output remains greater than 3 L/day for 1 week before the urea nitrogen returns to normal. Which of the following renal lesions was most likely present in this patient?

□ (A) Malignant nephrosclerosis 
□ (B) Renal vein thrombosis
□ (C) Membranous glomerulonephritis 
□ (D) Acute pyelonephritis
□ (E) Acute tubular necrosis
A

(E) This patient sustained muscle injury that resulted in myoglobinemia and myoglobinuria. The large amount of excreted myoglobin produces a toxic acute tubular necrosis. With supportive care, the tubular epithelium can regenerate, and renal function can be restored. During the recovery phase of acute tubular necrosis, patients excrete large volumes of urine because the glomerular filtrate cannot be adequately reabsorbed by the damaged tubular epithelium. Trauma is not a cause of malignant hypertension. A bilateral renal vein thrombosis is uncommon. Glomerulonephritis does not occur as a result of trauma. An infection with pyelonephritis is unlikely to be characterized by such a short course or such a marked loss of renal function.

63
Q

A 45-year-old man has had headaches, nausea, and vomiting that have worsened over the past 5 days. He has started “seeing spots” before his eyes. On physical examination, his blood pressure is 268/150 mm Hg. Urinalysis shows 1+ proteinuria; 2+ hematuria; and no glucose, ketones, or leukocytes. The serum urea nitrogen and creatinine levels are elevated. Which of the following histologic findings is most likely to be seen in this patient’s kidneys?

□ (A) Nodular glomerulosclerosis 
□ (B) Segmental tubular necrosis 
□ (C) Hyperplastic arteriolosclerosis
□ (D) Mesangial IgA deposition 
□ (E) Glomerular crescents
A

(C) This patient has malignant hypertension, which may follow long-standing benign hypertension. Two types of vascular lesions are found in malignant hypertension. Fibrinoid necrosis of the arterioles may be present; in addition, there is intimal thickening in interlobular arteries and arterioles, caused by proliferation of smooth muscle cells and collagen deposition. The proliferating smooth muscle cells are concentrically arranged, and these lesions, called hyperplastic arteriolosclerosis, cause severe narrowing of the lumen. The resultant ischemia elevates the renin level, which further promotes vasoconstriction to potentiate the injury. Nodular glomerulosclerosis is a feature of diabetes mellitus that slowly progresses over many years. Segmental tubular necrosis occurs in ischemic forms of acute tubular necrosis. An IgA nephropathy involves glomeruli, but not typically the interstitium or vasculature. Glomerular crescents are a feature of a rapidly progressive glomerulonephritis; however, the blood pressure elevation is not as marked as that seen in this patient.

64
Q

A 62-year-old man has had several episodes of hematuria over the past week. He has not experienced increased urinary frequency or dysuria. On physical examination, there are no remarkable findings. Urinalysis shows 4+ hematuria. The urine culture is negative. A cystoscopy is performed, and a 2-cm sessile, friable mass is seen on the right bladder wall. A biopsy specimen is obtained; the microscopic appearance is shown in the figure. Which of the following risk factors is most important in the pathogenesis of this bladder lesion?

□ (A) Smoking cigarettes
□ (B) Schistosomiasis
□ (C) Diabetes mellitus
□ (D) Chronic bacterial cystitis 
□ (E) Nodular prostatic hyperplasia
A

(A) The figure shows a high-grade urothelial carcinoma of the bladder. Cigarette smoking is the greatest risk factor in half of all men with such cancers. Schistosomiasis also is a risk factor for bladder cancer, although typically for squamous cell carcinoma. The increased risk of infection that occurs in diabetes mellitus and prostatic hyperplasia, with the resultant acute and chronic cystitis, does not predispose to urothelial carcinoma.

65
Q

A 38-year-old woman sees her physician because she has been feeling tired and lethargic for several months. On physical examination, she is afebrile, and her blood pressure is 140/90 mm Hg. Laboratory findings show hemoglobin, 10.3 g/dL; hematocrit, 30.9%; platelet count, 310,700/mm3; and WBC count, 5585/mm3. The serum creatinine level is 5.8 mg/dL. C3 nephritic factor is present in serum, and the ANA test result is negative. Urinalysis shows 2+ proteinuria. A renal biopsy is done; microscopic examination shows hypercellular glomeruli and prominent electron-dense deposits along the lamina densa of the glomerular basement membrane. Which of the following forms of glomerulonephritis is most likely to be present in this patient?

□ (A) Postinfectious glomerulonephritis
□ (B) Rapidly progressive glomerulonephritis
□ (C) Membranoproliferative glomerulonephritis
□ (D) Chronic glomerulonephritis
□ (E) Membranous glomerulonephritis

A

(C) This patient has type II membranoproliferative glomerulonephritis, or dense-deposit disease, which usually leads to chronic renal failure. Postinfectious glomerulonephritis is often characterized by a hypercellular glomerulus with infiltration of polymorphonuclear leukocytes, but no basement membrane thickening. A rapidly progressive glomerulonephritis is marked by crescents. The term chronic glomerulonephritis often is used when sclerosis of many glomeruli is present with no clear cause. Membranous glomerulonephritis is characterized by thickening of only the basement membrane and small electron-dense deposits.

66
Q

A 60-year-old man presents with a feeling of fullness in his abdomen and a 5-kg weight loss over the past 6 months. Physical examination is normal. Laboratory studies show hemoglobin of 8.2 g/dL, hematocrit of 24%, and MCV of 70 μm3. Urinalysis shows 3+ hematuria, but no protein, glucose, or leukocytes. Abdominal CT scan shows an 11-cm mass in the upper pole of the right kidney. A right nephrectomy is performed, and on gross examination the mass invades the renal vein. Microscopic examination of the mass shows cells with abundant clear cytoplasm. Which of the following molecular abnormalities is most likely to be found in tumor cell DNA?

□ (A) Homozygous loss of the von Hippel–Lindau (VHL) gene
□ (B) Mutational activation of the MET proto-oncogene
□ (C) Trisomy of chromosome 7 associated genes
□ (D) Integration of human papillomavirus-16 (HPV-16)
□ (E) Microsatellite instability

A

(A) Clear cell carcinoma, the most common form of kidney cancer, often manifests with painless hematuria, most often in individuals in the sixth or seventh decade. Approximately 80% of sporadic clear cell carcinomas show loss of both alleles of the VHL gene. Germline inheritance of the VHL mutation can give rise to von Hippel–Lindau syndrome, with peak incidence of renal cell carcinoma in the fourth decade, and they may have other tumors, including cerebellar and retinal hemangioblastomas and adrenal pheochromocytomas. Mutation of the MET gene on chromosome 7 is associated with the papillary variant of renal cell carcinoma. HPV-16 infection is associated with carcinomas of the uterine cervix. Microsatellite instability is a feature of the Lynch syndrome, also called hereditary nonpolyposis colon cancer syndrome, characterized by right-sided colon cancer and, in some cases, endometrial cancer.

67
Q

A 45-year-old Hispanic man has had increasing malaise with headaches and easy fatigability for the past 3 months. Physical examination reveals his blood pressure is 200/100 mm Hg. There are no palpable abdominal masses and no costovertebral tenderness. Laboratory studies show hemoglobin, 9.5 g/dL; hematocrit, 28.3%; MCV, 92 μm3; creatinine, 4.5 mg/dL; and urea nitrogen, 42 mg/dL. Urinalysis reveals 3+ hematuria and 3+ proteinuria, but no glucose or leukocytes. A renal biopsy is done; light microscopic examination of the biopsy specimen shows that approximately 50% of the glomeruli appear normal, but the rest show that a portion of the capillary tuft is sclerotic. Immunofluorescence staining shows IgM and C3 deposition in these sclerotic areas. Past history is significant for repeated episodes of passing dark brown urine, which failed to respond to corticosteroid therapy. Which of the following mechanisms is most likely responsible for his disease?

□ (A) Deposition of immune complexes containing microbial antigens
□ (B) Dysfunction of the podocyte slit diaphragm apparatus
□ (C) Deposition of anti–glomerular basement membrane antibodies
□ (D) Inherited defect in the basement membrane collagen
□ (E) Deposition of C3 nephritic factor (C3NeF)

A

(B) Corticosteroid-resistant hematuria and proteinuria in a Hispanic man leading to hypertension and renal failure is typical for focal segmental glomerulosclerosis (FSGS). FSGS is now the most common cause of nephrotic syndrome in adults in the United States. Specialized extracellular areas overlying the glomerular basement membrane between adjacent foot processes of podocytes are called slit diaphragms, and these exert control over glomerular permeability. Mutations in genes affecting several proteins, including nephrin and podocin, have been found in inherited cases of FSGS, and their dysfunction, possibly caused by cytokines or unknown toxic factors, is believed to be responsible for acquired cases of FSGS. FSGS also is seen in patients with HIV-associated nephropathy. Immune complexes containing microbial antigens cause postinfectious glomerulonephritis. Anti–glomerular basement membrane antibodies are responsible for Goodpasture syndrome. Inherited defects in basement membrane collagen cause Alport syndrome, also characterized by hematuria, but other congenital abnormalities, such as deafness, are often present, and nephrotic syndrome is uncommon. C3NeF is an autoantibody directed against C3 convertase, and it is seen in membranoproliferative glomerulonephritis type II.